Download as pdf or txt
Download as pdf or txt
You are on page 1of 312

1 March 2023 1

The first hydrogen plant in North East India will be set up


in which place in Assam

A. Sivasagar
B. Tezpur
C. Guwahati
D. Jorhat
Ans. D. Jorhat
1 March 2023 2

Who is known as the Luit Konwar of Assam


A. Binanda Chandra Baruah
B. Rudra Baruah
C. Dimbeswar Neog
D. None of the above
Ans. B. Rudra Baruah
1 March 2023 3

The first modern Assamese theatre hall was


established at

A. Sivasagar
B. Jorhat
C. Gauhati
D. Tezpur
Ans. D. Tezpur
1 March 2023 4

“Kathonibari Ghat” a collection of short stories was


written by

A. Mahim Bora
B. Mamoni Raisom Goswami
C. Nilomani Phukan
D. Nirupoma Borgohain
Ans. A. Mahim Bora
1 March 2023 5

Consider the following statements regarding BharatNet project


1.It will connect all the major cities of the country
2.It is the world’s second largest rural connectivity scheme
3.It will be implemented by Bharat Broadband network limited
4.It will be implemented by BSNL through collaboration with Airtel.
Which among the following statements is correct
A. 1 and 4 It is the world’s largest rural
connectivity scheme to be connected
B. 1,2 and 3
by the Optical Fibre network.
C. only 3
Implementing Agency: The project is
D. 2 and 3 being implemented by Bharat
Broadband Network Limited (BBNL)
Ans. C. Only 3
through a Special Purpose Vehicle
It is a project envisioned by the (SPV).
Government of India to digitally
Aim: To provide on-demand, affordable
connect all the Gram Panchayats (GPs)
broadband connectivity of 2 Mbps to 20
and Villages of India.
Mbps for all households of India
It originally aimed to provide broadband especially in rural areas.
services at 100 Mbps to around 2.5 lakh
gram panchayats of the country.
1 March 2023 6

Which of the following is included as part of India’s GDP?

a) Economic activities of US embassy in India.


b) Economic activities of Indian embassy in USA
c) Economic activities of residents of India in international
waters

Which of the above is correct

A. Only A
B. Only C
C. All of the Above
D. None of the Above
Ans. B. Only C
1 March 2023 7

Consider the following statements about PM-Saubhagya Scheme


1. Its Objective is to provide energy access to all by last mile connectivity.
2. It only provides electricity connections to all remaining un-electrified
households in rural areas.
Which of the above statements is/are incorrect
A. 1 only • Providing access to electricity
to all un-electrified households in
B. 2 only
rural areas and poor households
C. Both 1 and 2 in urban areas.
D. Neither 1 or 2 Beneficiaries:
Ans. B. 2 Only • They are identified using Socio
Economic and Caste Census
It was launched in 2017 to ensure (SECC) 2011 data.
electrification of all households in the
• However, un-electrified
country in rural as well as urban areas.
households not covered under
Objectives: the SECC data would also be
provided electricity
• Achieving Universal Household
connections under the scheme
Electrification in the country,
by paying Rs 500.
through last mile connectivity.
1 March 2023 8

The book Gandhi and Anarchy was authored by

A. Dr. B.R Ambedkar


B. V D Savarkar
C. Chettur Sankaran Nair
D. Kapil Sibal

Ans. C. Chettur Sankaran Nair


1 March 2023 9

Consider the following statements


1. Madurai Malli has references in Sangam Literature
2. Madurai Malli has a Geographical Indication Tag
Which of the above statements is/are incorrect
A. Only 1 Madurai and the neighboring places
of Madurai like Theni, Sivaganga,
B. Only 2
Virudhunagar and Dindigul.
C. Both 1 and 2
It was given GI recognition in 2013.
D. Neither 1 nor 2
There are extensive references to the
Ans. D. Neither 1 nor 2 flower in Sangam literature.
It is the Jasmine flower, known for its
strong and powerful, mysteriously
attractive & seductive fragrance.
Because of its place of origin, it is
known as Madurai Malli.
It is mainly cultivated in the district of
1 March 2023 10

Consider the following statements


1. According to Laffer curve, tax revenues always increase with increase in
tax rate.
2. According to Philips curve, there exist inverse relationship between rate of
unemployment and rate of inflation.
3. Lorenz curve is a graph showing the degree of inequality in income in a
given population or an economy.
Which of the statements given above is/are correct?

A. 1 and 3 only
B. 3 only
C. 1 and 2 only
D. 2 and 3 only
Ans. D. 2 and 3 Only
1 March 2023 11

Consider the following statements regarding Buddhism


1. Pali was the dominant language of Mahayana Buddhism.
2. Sthaviravada is a sub sect under Mahayana Buddhism.
3. Mahavamsa is an important text associated with Theravada Buddhism.
4. Kosala and Magadha are the only two kingdoms visited by Gautam
Buddha.
Which of the statements given above is/are incorrect

A. 1 and 3 only
B. 1, 2 and 4 only
C. 1, 2 and 3 only
D. 3 and 4 only
Ans. D. 3 and 4 Only
1 March 2023 12

Who was the last ruler of the Pala Dynasty

A. Brahmapala
B. Jayapala
C. Dharmapala
D. Harshapala
Ans. B. Jaya Pala
1 March 2023 13

Arrange these Harappan sites in order of their occurrence from


North to South
i. Kot Diji
ii. Kalibangan
iii. Harappa
iv. Surkotada
Options:
A. IV, II, III, I
B. III, II, I, IV
C. III, IV, I, II
D. IV, II, I, III
Ans. B. III, II, I, IV
1 March 2023 14

Who was awarded the Sahitya Akademi Yuva Puraskar 2021


from Assam in Assamese language

A. Anuradha Sarma Pujari


B. Abhijit Bora
C. Gautam Daimary
D. Dr. Pitambar Dev Goswami
Ans. B. Abhijit Bora - Deuka Kobai Jay (Short Stories)
1 March 2023 15

On 5th April 2022, Stand-up India scheme has completed six years. Which
of the following statements is/are correct with respect to Stand-up India
Scheme?

A. The scheme aims at promoting entrepreneurship among people with


disabilities.
B. Loans under the scheme is available for only green field project.
C. It facilitates loans only for manufacturing activities.
D. The offices of RBI is designated Stand-Up Connect Centres (SUCC).
Ans. B. Loans under the scheme is available for only green field project.
The Ministry of Finance has extended the Standup India Scheme up to the
year 2025. It was launched in April 2016 to promote entrepreneurship at
the grass-root level focusing on economic empowerment and job creation.
Loans under the scheme are available for only Greenfield projects.
1 March 2023 16

Consider the following statements regarding electoral bonds:


1. It is a financial instrument for making donations to political parties.
2. The bonds are issued for a maximum limit of 1 crore.
3. The validity of bonds is fifteen days from the date of issuance.
4. Donor name is mentioned in the bonds
Which of the following is/are true?
A. 1 and 2 Only are valid for fifteen days from the date
B. 2, 3 and 4 Only of issuance.
C. 1 and 3 Only The bonds are available for purchase by
D. 1, 3 and 4 Only any person (who is a citizen of India or
incorporated or established in India) for
Ans. C. 1 and 3 Only a period of ten days each in the months
Electoral Bond is a financial instrument of January, April, July and October as
for making donations to political may be specified by the Central
parties. Government.
The bonds are issued in multiples of Rs. A person being an individual can buy
1,000, Rs. 10,000, Rs. 1 lakh, Rs. 10 lakh bonds, either singly or jointly with other
and Rs. 1 crore without any maximum individuals.
limit. Donor’s name is not mentioned on the
State Bank of India is authorised to bond.
issue and encash these bonds, which
1 March 2023 17

The United Nations General Assembly (UNGA) was scheduled to vote on April 7
on a draft resolution to suspend Russia from the United Nations Human Rights
Council(UNHRC) for its alleged war on Ukraine.Which of the following is/are
correct regarding UNHRC
a). The membership of the Council is based on allocations to regions across
the world to ensure geographical representation.
b). Resolutions passed by UNHRC is binding on the parties.
c). Its most important members are US, UK, China, Japan and Russia
d). Each elected member serves for a term of three years.
Which of the following is/are true?

A. a, b and d
B. b, c and d
C. a and d only
D. all of the above
1 March 2023 18

Which of the following statements is/are true regarding


Okhla Sewerage Treatment Plant?

A. It is expected to play a major role in the cleaning the


Ganga river.
B. It will be the largest sewage removal plant of Asia.
C. It is located in Uttar Pradesh
D. Both a and b

Ans. B. It will be the largest sewage removal plant of Asia


1 March 2023 19

Which of the following pairs are correctly match


Book Author
1) Antim Jatra Sashi Sarma
2) Asomiya Akhor Jotoni Golak Chandra
Goswami
3) Hepahar Kathiya Bhaba Goswami
4) Megh Ghora Gautam Daimary

A. 1 and 2 only
B. 1, 2 and 3 only
C. 2 and 4 only
D. 1, 3 and 4 only
Ans. B. 1, 2 and 3 Only
1 March 2023 20

Who has been nominated from Assam to attend COP26 with


observer status ?

A. Disha Sarkar
B. Jadav Payeng
C. Moharana Choudhury
D. Rituraj Phukan
Ans. A. Disha Sarkar
1 March 2023 21

Assam’s first Miyawaki Forest Projects is


launched at

A. Kaziranga National Park


B. Guwahati University
C. Raimona National Park
D. Assam Down Town University
Ans. . D. Assam Down Town University, Panikhaiti
1 March 2023 22

Karnaphuli river originates in which of the following


states

A. Nagaland
B. Meghalaya
C. Manipur
D. Mizoram
Ans. D. Mizoram
1 March 2023 23

Yuvika, recently seen in news is an initiative of

A. UGC
B. ISRO
C. RBI
D. DRDO
Ans. B. ISRO
1 March 2023 24

Which of the following profession is emerging as Go-To


profession in Assam

A. Handloom
B. Brass and Bell metal industry
C. Fish Farming
D. Agriculture

Ans. C. Fish Farming


1 March 2023 25

Fasal Bima Pathshala is an initiative of

A. Ministry of Agriculture & farmers’ Welfare


B. Ministry of Commerce & Industry
C. Ministry of Finance
D. NITI Aayog
Ans. A. Ministry of Agriculture & farmers’ Welfare
1 March 2023 26

Which the of the following state launched the Project


BOLD

A. Assam
B. Rajasthan
C. Arunachal Pradesh
D. Madhya Pradesh
Ans. B. Rajasthan
1 March 2023 27

Who was awarded the Sahitya Akademi Yuva Puraskar


2021 from Assam for poetry in Bodo language

A. Ratneswar Narzary
B. Rituraj Basumatary
C. Gautom Daimary
D. Dwijen Das

Ans. C. Gautom Daimary


1 March 2023 28

Recently seen in news, DigiSaathi is an initiative of

A. UGC
B. Indian Railways
C. RBI
D. SEBI
Ans. C. RBI
1 March 2023 29

Which of the following cities is located Asia’s largest Bio-


CNG plant

A. Guwahati
B. Mumbai
C. Indore
D. Chennai
Ans. C. Indore
1 March 2023 30

Recently seen in the news Dapivirine, is

A. Covid-19 Vaccine
B. Antiretroviral drug
C. Vaccine for Malaria
D. Ebola drug
Ans. B. Antiretroviral drug
1 March 2023 31

The author of Asomiya Sahitya Buranji is

A. Nathan Brown
B. Jatindranath Duara
C. Gunabhiram Barua
D. Devendranath Bezbarua
Ans. D. Devendranath Bezbarua
1 March 2023 32

Prachya Sasanvali was edited by

A. Dr. Maheswar Neog


B. Dr. Birinchi Kumar Barua
C. Dr. Surya Kummar Bhuyan
D. Gunabhiram Barua
Ans. A. Dr. Maheswar Neog
1 March 2023 33

Which of the following Ministry launched Mera Ration


App

A. Ministry of Consumer Affairs


B. Ministry of Agriculture and farmers welfare
C. Ministry of Rural Development
D. Ministry of Home Affairs
Ans. A. Ministry of Consumer Affairs
1 March 2023 34

Atmanirbhar Niveshak Mitra Portal will help which of the


following investors

A. Start up investors
B. Foreign Direct Investors
C. Foreign Institutional Investors
D. Domestic Investors
Ans. D. Domestic Investors
1 March 2023 35

Which tribe of North East India celebrates the Agalmaka


festival

A. Mising
B. Angami
C. Garo
D. Kuki
Ans. C. Garo
1 March 2023 36

Which of the following state has allowed sale of Mahua


as a liquor heritage?
A. Nagaland
B. Madhya Pradesh
C. Himachal Pradesh
D. Manipur
Ans. B. Madhya Pradesh
1 March 2023 37

Who has been awarded as the Best Youth Artist Award at


the Toronto International Women Film Festival 2022

A. Shivarani Kalita
B. Nayanmoni Mura
C. Bhonita Das
D. Kunjalata Boro
Ans. B. Nayanmoni Mura
1 March 2023 38

India Cycles4Change Challenge is an initiative of

A. Ministry of Agriculture & farmers’ Welfare


B. NITI Aayog
C. Ministry of Youth Affairs and Sports
D. Ministry oh Housing and Urban Affairs
Ans. D. Ministry oh Housing and Urban Affairs
1 March 2023 39

Which of the following Assamese film has been selected


to be screened at the 18th Indian Film Festival Stuttgart

A. Bridge
B. Ronuwa
C. Porichoy
D. God on the Balcony
Ans. D. God on the Balcony
1 March 2023 40

A reserve forest is an area with complete protection


constituted according to the

A. Indian Wildlife Act 1972


B. Indian Forest Act 1927
C. National Forest Policy, 1952
D. None of the Above

Ans. B. Indian Forest Act 1927


1 March 2023 41

Recently discovered species Litoria mira is a


A. Amphibian
B. Mammal
C. Reptiles
D. None of the Above
Ans. A. Amphibian
1 March 2023 42

The term Unicorn is used to describe a start up company with a


value of over

A. $ 100 billion
B. $ 10 billion
C. $ 5 billion
D. $ 1 billion
Ans. D. $ 1 billion
1 March 2023 43

The Emission Gap Report is released by

A. UNEP
B. UNFCCC
C. WEF
D. GEF
Ans. A. UNEP
1 March 2023 44

Consider the following statements regarding High Court


I. Indian Constitution has provisions that proceeding of
every High Court shall be in English language.
II. The President of India may authorize the use of the
Hindi language or any other language in the
proceeding of a High Court.
Select the correct statements using the codes

A. I only
B. II only
C. Both I and II
D. Neither I nor II
Ans. A. I only
1 March 2023 45

Which of the following bank in India has launched the


country’s first credit card made from recycled PVC Plastic
A. ICICI
B. HDFC
C. HSBC
D. SBI
Ans. C. HSBC
1 March 2023 46

World Press Freedom Index is released by

A. Transparency International
B. Reporters Without Borders
C. Freedom House
D. International Federation of Journalists
Ans. B. Reporters Without Borders
1 March 2023 47

Consider the following statements


1. National Technology Day is celebrated every year
in India on May 11
2. The theme of National Technology Day 2022 is
Science and Technology for a Sustainable Future
Select the correct statements
A. 1 Only Future.
B. 2 only Theme 2022- Integrated
C. Both 1 and 2 Approach in science and
D. Neither 1 nor 2 technology for a sustainable
future
Ans. A. 1 Only
Theme 2021- Science and
Technology for a Sustainable
1 March 2023 48

Pampa Sagar Dam is located on which of the following


rivers?

A. Bhima
B. Kosi
C. Palar
D. Tungabhadra
Ans. D. Tungabhadra
1 March 2023 49

Recently seen in news Cyclone Asani, is named by which


of the following countries
A. India
B. Sri Lanka
C. Myanmar
D. Bangladesh

Ans. B. Sri Lanka


1 March 2023 50

Recently in news, MACS1407 is related to

A. Rice
B. Soybean
C. Tea
D. Cotton

Ans. B. Soybean
1 March 2023 51

Janjatiya Gaurav Divas is celebrated to commemorate the


birth anniversary of

A. Birsa Munda
B. Sidhu Muru
C. Alluri Sita Rama Raju
D. None of the Above
Ans. A. Birsa Munda
1 March 2023 52

Dengue fever is caused by a parasite which is a

A. Virus
B. Bacteria
C. Protozoa
D. None of the above
Ans. A. Virus
1 March 2023 53

Which of the following organs of human body affects by


Hepatitis B

A. Kidney
B. Liver
C. Pancreas
D. Both A and B
Ans. B. Liver
1 March 2023 54

𝑊ℎ𝑖𝑐ℎ 𝑜𝑓 𝑡ℎ𝑒 𝑓𝑜𝑙𝑙𝑜𝑤𝑖𝑛𝑔 𝑠𝑡𝑎𝑡𝑒 𝑖𝑛 𝐼𝑛𝑑𝑖𝑎 𝑏𝑒𝑐𝑎𝑚𝑒 𝑡ℎ𝑒


𝑓𝑖𝑟𝑠𝑡 𝑠𝑡𝑎𝑡𝑒 𝑡𝑜 𝑑𝑒𝑣𝑒𝑙𝑜𝑝 10𝐺𝑊 𝑠𝑜𝑙𝑎𝑟 𝑝𝑜𝑤𝑒𝑟 𝑐𝑎𝑝𝑎𝑐𝑖𝑡𝑦

A. Assam
B. Gujarat
C. Rajasthan
D. Karnataka
Ans. C. Rajasthan
1 March 2023 55

Millennium Challenge Corporation (MCC) is an initiative


of which of the following countries?

A. India
B. USA
C. Japan
D. China
Ans. B. USA
1 March 2023 56

Recently, India commenced Exercise Bongo Sagar with


which of the following countries?

A. Bhutan
B. Bangladesh
C. Myanmar
D. None of the above
Ans. B. Bangladesh
1 March 2023 57

The South East Asia Ramayana Research Institute,


was started by

A. Lakshminath Bezbaruah
B. Indira Goswami
C. Bhabendra Nath Saikia
D. Bishnu Prasad Rabha
Ans. B. Indira Goswami
1 March 2023 58

Which of the following temples is also known as the


White Pagoda ?

A. Konark Sun Temple


B. Lingaraja Temple
C. Jagannath Temple Dibrugarh
D. None of the Above
Ans. D. None of the above
Jagannath temple of Puri was called the “White Pagoda.
The temple is believed to be constructed in
the 12th century by King Anatavarman Chodaganga Deva
of the Eastern Ganga Dynasty.
1 March 2023 59

Portal Santusht is under the aegis of which of the


following ministries
A. Ministry of Labour and Employment
B. Ministry of Health and Family Welfare
C. Ministry of Human Resource Development
D. Ministry of Corporate Affairs
Ans. A. Ministry of Labour and Employment
1 March 2023 60

Who won the First Assam Valley Literary Award

A. Homen Borgohain
B. Bhabendra Nath Saikia
C. Syed Abdul Malik
D. Nabakanta Barua

Ans. B. Bhabendra Nath Saikia


1 March 2023 61

Kanaklata Barua was related with which one of the


following movement

A. Non- Cooperation Movement


B. Quit India Movement
C. Civil Disobedience Movement
D. None of the above
Ans. B. Quit India Movement
1 March 2023 62

Dampa Tiger reserve is located in which of the


following states

A. Arunachal Pradesh
B. Mizoram
C. Tripura
D. Manipur
Ans. B. Mizoram
1 March 2023 63

Recently, Which of the following NGO published the State of


India’s Environment Report 2021

A. Global Footprint Network


B. Centre for Science and Environment
C. Bombay Natural History Society
D. Centre for Environment Education
Ans. B. Centre for Science and Environment
1 March 2023 64

Through which of the following Acts, the Governor-


General of Bengal came to be called as the Governor
General of India

A. The Charter Act of 1793


B. The Charter Act of 1813
C. The Charter Act of 1833
D. The Charter Act of 1853
Ans. C. The Charter Act of 1833
1 March 2023 65

Consider the following statements


1. “Halodhiya Choraye Baodhan Khai” won the National
Film Award for Best Feature Film in the year 1988.
2. The film was directed by Jahnu Barua.
Which of the statements given above is/are correct

A. 1 only
B. 2 only
C. Both 1 and 2
D. Neither 1 nor 2
Ans. C. Both 1 and 2
1 March 2023 66

Recently, Which of the following organisation released


Footing the Bill report

A. FAO
B. Oxfam International
C. WHO
D. The Organisation for Economic Co-Operation and
Development
Ans. B. Oxfam International
1 March 2023 67

Recently, seen in news Build Back Better World


(B3W) initiative proposed by

A. World Bank
B. WEF
C. G7
D. None of the above
Ans. C. G7
1 March 2023 68

Snake island recently seen in news, is located in which of


the following seas

A. Black Sea
B. South China Sea
C. Caspian Sea
D. None of the Above
Ans. A. Black Sea
1 March 2023 69

Who was the first Assamese Chief Justice of the Gauhati


High Court?

A. Justice Parbati Kumar Goswami


B. Justice Holiram Deka
C. Harishankar Brahma
D. Justice K.N Saikia
Ans. B. Justice Holiram Deka
1 March 2023 70

Consider the following countries regarding members of I2U2


grouping
1. India
2. UK
3. Israel
4. Ukraine
Select the correct answer using the code

A. 1 and 2 only
B. 2 and 3 only
C. 1 and 3 only
D. 2 and 4 only
Ans. C. 1 and 3 Only
I2U2 is the new grouping formed by four nations- India, Israel, UAE,
and the US.
1 March 2023 71

The novel Seuji Pataar Kahani, is based on the life in a tea


garden of Assam written under the pseudonym Rasna
Barua in 1955. What is the author’s real name

A. Kanchan Baruah
B. Biren Barkotoki
C. Nirupoma Borgohain
D. Birinchi Kumar Baruah
Ans. D. Birinchi Kumar Baruah
1 March 2023 72

Who is the first woman to receive Sahitya Akademi award

A. Amrita Pritam
B. Ashapurna Devi
C. Mahasweta Devi
D. None of the above
Ans. B. Ashapurna Devi
1 March 2023 73

Who was the first Foreign Minister of free India

A. Gulzari Lal Nanda


B. Jawaharlal Nehru
C. Dr. John Mathai
D. Maulana Abul Kalam Azad
Ans. B. Jawaharlal Nehru
1 March 2023 74

With reference to Wildlife Sanctuaries/ National Parks/


Wetlands, recently seen in news Which of the following
pairs is correctly matched
1. Sultanpur National Parks- Rajasthan
2. Pala Wetland- Mizoram
3. Bakhira Wildlife Sanctuary- Uttar Pradesh
4. Meghamalai Wildlife Sanctuary- Tamil Nadu
A. 1, 2 and 3 only
B. 1, 3 and 4 only
C. 2, 3 and 4 only
D. 2 and 4 only
Ans. C. 2, 3 and 4 Only
1 March 2023 75

Which of the following state is known as


Molasses Basin

A. Meghalaya
B. Nagaland
C. Mizoram
D. Manipur

Ans. C. Mizoram
1 March 2023 76

The Swaraj Party was organised by


A. Mahatma Gandhi and Motilal Nehru
B. Annie Besant and B.G Tilak
C. Lala Lajapat Rai and B.G Tilak
D. Chittaranjan Das and Motilal Nehru

Ans. D. Chittaranjan Das and Motilal Nehru


1 March 2023 77

Chemayungdung Glacier is the source of which


of the following rivers

A. Brahmaputra
B. Indus
C. Sutlej
D. Beas

Ans. A. Brahmaputra
1 March 2023 78

The first effort at drafting a Dominion Status


Constitution for India was made in response to
the
A. August Offer
B. Simon Commission
C. First Round Table Conference
D. Government of India Act 1919

Ans. B. Simon Commission


1 March 2023 79

Consider the following statements regarding Orunodoi


magazine
1. Orunodoi is the first Assamese language
magazine published in 1849.
2. The first Assamese romantic poem was published
in the magazine.
Select the correct statements using the codes

A. 1 only
B. 2 only
C. Both 1 and 2
D. Neither 1 nor 2
Ans. D. Neither 1 nor 2
1 March 2023 80

Who among the following did not attend the


First Round Table Conference?

A. Sir Tej Bahadur Sapru


B. Dr B. R. Ambedkar
C. M. K. Gandhi
D. Both (A) and (C)

Ans. C. M.K. Gandhi


1 March 2023 81

The Gender Parity Index is released by which of


the following agencies
A. UNDP
B. UNESCO
C. WEF
D. UN Commission on Women

Ans. B. UNESCO
1 March 2023 82

Which city of Assam is known as the City of Eternal


Romance

A. Sivasagar
B. Jorhat
C. Gauhati
D. Tezpur
Ans. D. Tezpur
1 March 2023 83

Who appoints the Acting Chief Justice for the high


courts?
A. CJI in consultation with President
B. Governor
C. President
D. None of the above
Ans. C. President
1 March 2023 84

Which of the following leaders died as a result


of Injuries sustained during a protest
demonstration against Simon Commission?

A. Bal Gangadhar Tilak


B. Goplakrishna Gokhale
C. LaIa Lajpat Rai
D. None of the above

Ans.C. LaIa Lajpat Rai


1 March 2023 85

Who gave the slogan “Inquilab Zindabad”?

A. Chandra Shekhar Azad


B. Bhagat Singh
C. Subhash Chandra Bose
D. None of the above

Ans. D. None of the above


Maulana Hasrat Mohani gave the slogan “Inquilab
Zindabad”
1 March 2023 86

Manipuri Princess Kuranganayani was married


to which Ahom King?

A. Rajeswar Singha
B. Gaurinath Singha
C. Udayaditya Singha
D. None of the above
Ans. A. Rajeswar Singha
1 March 2023 87

The word 'socialist secular' and 'unity and


integrity of the Nation' were added to the
Constitution of India by
A. 42nd Amendment
B. 44th Amendment
C. 46th Amendment
D. None of the above

Ans. A. 42ndAmendment
1 March 2023 88

Which of the following Ahom king crushed the


rebellion of the Baro Bhuyans?
A. Rudra Singha
B. Jayadhwaj Singha
C. Pratap Singha
D. Chakradhwaj Singha
Ans. C. Pratap Singha
1 March 2023 89

Subhash Chandra Bose established the


Provisional Government of Free India at?

A. Singapore
B. Berlin
C. Tokyo
D. None of the above

Ans. A. Singapore
1 March 2023 90

The ideal of ‘Welfare state’ in the Indian


constitution is enshrined in its
A. Preamble
B. Directive Principles of State Policy
C. Fundamental Rights
D. None of the above

Ans. B. Directive Principles of State Policy


1 March 2023 91

The “Deodhani” dance is associated with


the worship of which Goddess?

A. Durga
B. Manasa
C. Parvati
D. None of the above
Ans.B. Manasa
1 March 2023 92

Which of the following committee was constituted


to review the implementation of Fiscal Responsibility and
Budget Management Act, 2003?

A. Raghuram Rajan Committee


B. Arvind Subramanian Committee
C. Urjit Patel Committee
D. None of the above
Ans. D. None of the above
The Fiscal Responsibility and Budget Management (FRBM) Act
was enacted in 2003 which set targets for the government to
reduce fiscal deficits.In May 2016, the government set up a
committee under NK Singh to review the FRBM Act.
1 March 2023 93

Which of the following is the


largest salix swamp forest in North-East India?

1. Dibru-Saikhowa National Park


2. Dehing Patkai National Park
3. Nameri National Park
4. Raimona National Park
Ans. 1. Dibru-Saikhowa Naional Park
1 March 2023 94

Q: ‘Formosa Strait’ separates which of the


following land masses?

A. Italy and Cuba


B. India and Sri Lanka
C. Mainland China and Taiwan
D. None of the above
Ans. C. Mainland China and Taiwan
Formosa Strait also called ‘Taiwan Strait’ separates the
coast of China’s Fukien province and the island of Taiwan.
It has been in the news recently in the context of the
ongoing conflict between China and Taiwan. The median
line in the Formosa Strait functions as an unofficial
border between Taiwan and China that has
been respected by both sides for the past few decades.
1 March 2023 95

The daughter of which Varman king was married


to the Prince of Kashmir ?
A. Pushya Varman
B. Samudra Varman
C. Bala Varman
D. Sthita Varman
Correct Option: C
Bala Varman ruled Kamarupa during the period 398–422
CE. He organized Svayamavara for his
daughter Kalyanavarman and the event was attended by
princes of different kingdoms. The Princess eventually
accepted the prince of Kashmir Meghavana as her groom.
Detailed account of this event can be found
in Rajtarangini: the legendary and historical chronicle of
the north-western Indian subcontinent, particularly the
kings of Kashmir, written by Kashmiri historian Kalhana in
the 12th century CE.
1 March 2023 96

Federal Court of India was established under


the provision of
A. Councils act, 1891
B. Indian Council act, 1909
C. Government of India Act, 1919
D. Government of India Act, 1935
Correct Option: D
The federal court of India was judicial body,
established in India in 1937 under the provisions of
the Government of India Act, 1935. It
functioned until the supreme court of India was
established in 1950
1 March 2023 97

Which of the following statement/s is/are incorrect with reference to


the Revolt of 1857?
1.The revolt began with the rebellion of the Company troops at
Barrackpore
2.The seizure of rent free estates was a factor behind support by
religious orthodoxy
3.The revolt received widespread support among the educated
classes
4.The rebel leader Bakht Khan led the rebellion in Delhi
Options:
A. I, II, IV
B. I and III
C. III and IV
D. III Only
Ans. B. I and III
1 March 2023 98

Which of the following organization releases


the World Press Freedom Index ?
A. World International Reporters Organization
B. International Telecommunication Organization
C. Reporters without Borders
D. None of the Above
Ans. C. Reporters without Borders
The Paris based Reporters Without Borders (RSF)
publishes annually a World Press Freedom Index (WPFI)
purporting to evaluate the level of freedom available to
the media in 180 countries. As per World Press Freedom
Index 2022, India's ranking dropped to 150th position
from last year's 142nd rank.
1 March 2023 99

The Ajnala Massacre of 1857, recently in


news, was perpetrated at:
A. Gwalior
B. Kanpur
C. Awadh
D. Punjab
Correct Option: D
Recently, researchers concluded that the skeleton
remains of more than 200 long-dead men found in a
well beneath a gurudwara in Ajnala, on the outskirts
of Amritsar, belonged to soldiers massacred by the
British in 1857. Watch further analysis on
our YouTube Channel.
1 March 2023 100

The notion of ‘Freedom’, in a society, implies


1.Absence of external constraints
2.All decisions are made collectively
3.Conditions in which people can develop their abilities
Select the correct answer using the codes below.

A. 1 and 2 only
B. 2 and 3 only
C. 3 only
D. 1 and 3 only
Correct Option: D
Freedom is said to exist when external constraints on the individual
are absent. Freedom is also about expanding the ability. of people
to freely express themselves and develop their potential. Freedom,
in this sense, is the condition in which people can develop their
creativity and capabilities.
So, A free society would be one which enables all its members to
develop their potential with the minimum of social constraints.
1 March 2023 101

Which of the following are the salient features of


Indian Constitution?
1.Federal System with Unitary Bias
2.Drawn From Various Sources
3.Integrated and Independent Judiciary
4.Two-tier Government
Select the correct answer using the code given below
A. 1, 2 and 4 only
B. 2 and 3 only
C. 1, 2 and 3 only
D. 3 and 4 only
Correct Option: C
Point 4 is not correct. The constitution of India provides
three tiers of governance: 1. Central Government. 2.
State Government. 3. Panchayati Raj
1 March 2023 102

Recently seen in news, Harimau Shakti


Exercise is between India and which nation
A. Indonesia
B. Bangladesh
C. Malaysia
D. France
Correct Option: C
India held a major combat exercise recently called
Exercise ‘Harimau Shakti’ with Malaysia from April 30 to
May 13. The exercise, was held in the dense forests
of Sengai Perdik, Hulu Langat, Malaysia, and was aimed at
“bolstering cooperation and coordination” between the
armed forces of the two countries. This initiative is driven
by India’s ‘Act East Policy’ that also involves a
steady build up of military ties with ASEAN countries.
1 March 2023 103

India hosted the Industry Transition Dialogue


with which country

A. Sweden
B. France
C. Australia
D. Sri Lanka
Correct Option: A
Industry Transition Dialogue was hosted by India and
Sweden on 1 June 2022 in Stockholm as a part of their
joint initiative Leadership for Industry Transition (LeadIT).
This high level of dialogue has contributed to the UN
Conference ‘Stockholm+50’: a healthy planet for the
prosperity of all –our responsibility, our opportunity’, that
took place on 2 and 3 June 2022 and set the agenda
for COP27.
1 March 2023 104

Arrange the following in their proper chronological order:


1.Second Urbanisation of India
2.Aryan Migration
3.Gandhara School of Art
4.Battle of the ten Kings
Options:

A. iv, ii, i, iii


B. ii, i, iv, iii
C. ii, i, iii, iv
D. ii, iv, i, iii
Correct Option: D
1 March 2023 105

Consider the following statements about Rafale Aircraft:


1.Rafale is a single engine aircraft
2.It is manufactured by Dassault Aviation of France
3.India has acquired 106 Rafale Aircrafts
4.The first squadron of Rafale is called the
Golden Daggers
Select the correct statements

A. a and c
B. a, c and d
C. Only b
D. All of the above
Correct Option: C. only b
1 March 2023 106

Which among the following are related to The Supreme Court’s landmark
decision in NALSA v. Union of India case?
A. Declaration of Right to privacy as the Fundamental Right
B. Abolition of triple talaq
C. Recognition that transgender persons have fundamental rights
D. 10% reservation to EWS
Correct Option: C
A case was filed by the National Legal Services Authority of India (NALSA) to
legally recognize persons who fall outside the male/female gender binary,
including persons who identify as “third gender”. In it’s landmark judgement in this
case, the Supreme Court affirmed that transgender and intersex persons have a
constitutional right to self-identify as male, female, or transgender without medical
intervention. The Court interpreted ‘dignity’ under Article 21 of the Constitution to
include diversity in self-expression, which allowed a person to lead a dignified life. It
placed one’s gender identity within the framework of the fundamental right to dignity
under Article 21. Further, it noted that the right to equality (Article 14 of the
Constitution) and freedom of expression (Article 19(1)(a)) was framed in gender-
neutral terms (“all persons”). Consequently, the right to equality and freedom of
expression would extend to transgender persons.
1 March 2023 107

What does venture capital mean? (UPSC 2014)

A. A short-term capital provided to industries.


B. A long-term start-up capital provided to new entrepreneurs
C. Funds provided to industries at times of incurring losses
D. Funds provided for replacement and renovation of Industries
Ans: B. A long-term start-up capital provided to new entrepreneurs
Venture capital is a form of fund for a new or growing business.
It usually comes from venture capital firms that specialize in
building high risk financial portfolios.
N.B. Recently, the government of India has stated that India ranks 3rd
globally in Start-up Ecosystem and also, in terms of the number
of Unicorns.
1 March 2023 108

The Multi-dimensional Poverty Index developed by Oxford Poverty and


Human Development Initiative with UNDP support covers which of the
following? (UPSC 2012)
1. Deprivation of education, health, assets and services at household level
2. Purchasing power parity at national level
3. Extent of budget deficit and GDP growth rate at national level
Select the correct answer using the codes given below:
A. 1 only
B. 2 and 3 only
C. 1 and 3 only
D. 1, 2 and 3

Ans. A. 1 only
The Multidimensional Poverty Index (MPI) reflects the
deprivations that a poor person faces simultaneously with respect
to education, health and living standards.
1 March 2023 109

Consider the following statements regarding ESZ’s


1. Eco Sensitive Zones are the areas that are declared under the Environment
(Protection) Act, 1986.
2. The Environment (Protection) Act, 1986 does not mention the word “Eco-Sensitive
Zones”.
Select the correct answer using the code given below

A. 1 only
B. 2 only
C. Both 1 and 2
D. Neither 1 nor 2
Ans. C. Both 1 and 2
ESZ/ ESA (Eco-Sensitive Areas) are declared as per Environment Protection
Act 1986.These were included in the National Wildlife Action Plan (2002-
2016).
The Environment (Protection) Act, 1986 does not mention the word “Eco
Sensitive Zones”.
The purpose of declaring ESZs is to create some kind of “shock absorbers” to the protected
areas by regulating and managing the activities around such areas.
1 March 2023 110

Consider the following statements regarding Cheetah


1. The cheetah is the world’s fastest land mammal.
2. The reintroduction of Cheetahs will help restore India’s open forests and
grassland ecosystems.
Select the correct answer using the codes given below
A. 1 only
B. 2 only
C. Both 1 and 2
D. Neither 1 nor 2
Ans. C. Both 1 and 2
The cheetah is the world’s fastest land mammal that lives in Africa and Asia.
The reintroduction of Cheetahs will help restore India’s open forests and
grassland ecosystems. Also having cheetahs will result in greater biodiversity,
And biodiversity is the hallmark of healthy ecosystems.
The Asiatic cheetah is classified as a “critically endangered” species by the IUCN Red
List.
1 March 2023 111

Consider the following statements regarding the term “State” defined under the
Article 12
1. Legislative organs of centre and states
2. Executive organs of centre and states
3. Local authorities
Select the correct answer using the codes given below
A. 1 only
B. 2 and 3 only
C. 1 and 3 only
D. 1, 2 and 3
Ans. D. 1, 2 and 3
The term “State” under Art. 12 includes
▪ The government and Parliament of India i.e. Executive and Legislature of the
Union
▪ The government and Legislature of each state i.e. Executive and Legislature of
State
▪ All local and other authorities i.e. panchayats etc
▪ All other authorities i.e. statutory or non statutory like LIC
1 March 2023 112

Match the following


Group-A Group-B
1. Ugratara Devalaya a. Rajeswar Singha
2. Sukleswar Devalaya b. Gadadhar Singha
3. Umananda Mandir c. Siva Singha
4. Negheriting Shiva Doul d. Pramatta Singha
A. 1-a, 2-d, 3-c, 4-b located on the Sukreswar or
Itakhuli hill on the South bank of
B. 1-d, 2-c, 3-b, 4-a
the river Brahmaputra, built by
C. 1-c, 2-d, 3-b, 4-a Pramatta Singha
D. 1-d, 2-a, 3-c, 4-b Umananda Mandir: Umananda Mandir
is a Shiva temple located at the
Ans. C Umananda Island in the middle or
Ugratara Devalaya: Ugratara Devalaya river Brahmaputra built by
is a temple dedicated to Ugratara Gadadhar Singha
located in the western side of JorNegheriting Shiva Doul: The doul is a
Pukhury tanks in the heart of Shiva temple at Dergaon in 1765
Guwahati city built by Ahom King it was re-constructed by Rajeswar
Siva Singh in 1725 AD Singha.
Sukleswar Devalaya: The temple is
1 March 2023 113

Which of the following personality is popularly known as


“Sugandhi Pokhilar Kobi”?

A. Hiren Gohain
B. Hiren Bhattacharya
C. Mahim Bora
D. Rudra Barua
Ans. B. Hiren Bhattacharya ➢ Binhinna Dinar Kabita- 1974
Popularly known as Hiru Da among ➢ Kabitar Rode- 1976
assamese, Also known as Sugandhi ➢ Tomar Gaan- 1976
Pokhilar Kobi. Hiren Bhattacharya was ➢ Sugandhi Pokihla ( Selected
poems)-1981
born in Jorhat district in 1932.
➢ Saichar Pathar Manuh- 1991
Published—
➢ Mor Desh Mor Premor Kabita-
1972
1 March 2023 114

Global Plastic Action Partnership is an initiative of?

A. World Trade Organization


B. World Economic Forum
C. International Cooperation on Plastic Pollution
D. None of the above
Ans. B. World Economic Forum
Maharashtra joined the World Economic
Forum's Global Plastic Action Partnership GPAP at its
Annual Meeting 2022 in Davos in order to promote
regional efforts to fight plastic pollution.
1 March 2023 115

First Assamese novel "Bhanumati" was written


by?
A. Padmanath Gohain Barua
B. Gunabhiram Barua
C. Lakshminath Bezbaruah
D. Syed Abdul Malik
Ans. A. Padmanath Gohain Barua

Bhanumoti or Bhanumati was the first


Assamese novel. Published in 1890,
Bhanumoti was written by Padmanath Gohain
Barua first president of Asam Sahitya Sabha.
1 March 2023 116

Consider the following statements regarding Barak river


1. The source of the river is Loktak Lake.
2. The river is one of the habitats of Ganges River Dolphin.
Select the correct statements using the codes given below
A. 1 Only
B. 2 Only
C. Both
D. Neither
Ans. B. 2 Only
The Barak River is a 900-kilometre-long river flowing through the
states of Manipur, Nagaland, Mizoram and Assam in India and into
the Bay of Bengal via Bangladesh. Barak river originates near Liyai
Khullen Village in Manipur state of India.
The endangered Ganges River Dolphin inhabits the Ganges-
Brahmaputra- Meghna (GBM) river basin of India, Nepal and
Bangladesh and the disjunct Karnaphuli-Sangu river basin of
Bangladesh. This includes the Barak river system.
1 March 2023 117

Consider the following statements regarding Haipou Jadonang


1. He was a Rongmei Naga leader from Nagaland.
2. He founded the Harakka army.
Select the correct statements using the codes given
below

A. 1 only
B. 2 only
C. Both
D. Neither
Ans. B. 2 Only
Haipou Jadonang, was a Rongmei Naga leader from
Nagaland. He founded the Harakka army. This included both
men and women, who were well trained in military strategy,
weapons and reconnaissance missions
1 March 2023 118

International Labour Organization’s Conventions 138 and 182 are related to

A. Child Labour
B. Adaptation of agricultural practices to global climate change
C. Regulation of food prices and food security
D. Gender parity at the workplace

Ans. A Child Labour

The eight-core conventions of the ILO are:

➢ Forced Labour Convention (No. 29)


➢ Abolition of Forced Labour Convention (No.105)
➢ Equal Remuneration Convention (No.100)
➢ Discrimination (Employment Occupation) Convention (No.111)
➢ Minimum Age Convention (No.138)
➢ Worst forms of Child Labour Convention (No.182)
➢ Freedom of Association and Protection of Right to Organized Convention (No.87)
➢ Right to Organize and Collective Bargaining Convention (No.98)
1 March 2023 119

Sima Dutta is the pseudonym of

A. Birendranath Dutta
B. Lila Gogoi
C. Homen Borgohain
D. Navakanta Baruah
Ans. D. Navakanta Baruah

He was also known as Ekhud Kokaideu. As Sima


Dutta he wrote many poems in his early life.
1 March 2023 120

In which one of the following states are Namchik


Namphuk coalfields located?
A. Arunachal Pradesh
B. Meghalaya
C. Manipur
D. Mizoram
Ans. A. Arunachal Pradesh
Namchik - Namphuk coal field is located in
Changlang district of Arunachal Pradesh.
1 March 2023 121

Where was the First Session of the India National


congress held in December 1885

A. Ahmedabad
B. Bombay
C. Calcutta
D. Delhi
Ans. B. Bombay
INC was formed on 28 Dec, 1885 in Bombay and its
first session was held under the presidentship of W.C
Banerjee.
1 March 2023 122

Consider the following statements regarding NABARD


1. It is a statutory body.
2. It was established on the recommendations of the
Narasimham Committee.
Select the correct statements using the codes given below
A. 1 only
B. 2 only
C. Both
D. Neither
Ans. A. 1 only
It is a statutory body established in 1982 under
Parliamentary act National Bank for Agriculture and Rural
Development Act, 1981. It was established on the
recommendations of the B.Sivaraman Committee.
1 March 2023 123

West Seti hydroelectric Project is related to which of the


following country?

A. India
B. Nepal
C. Bhutan
D. Myanmar
Ans. B. Nepal
West Seti hydroelectric Project is a proposed 750
megawatt Hydroelectric Project, which is to built on
the Seti River in the Far-Western Development
Region of Nepal.
1 March 2023 124

Which of the following are regarded as the main


features of the “Rule of Law” ?
1.Limitation of powers
2.Equality before law
3.Peoples’ responsibilities towards government
4.Liberty and Civil Rights
Select the correct answers using the code given below.

A. 1 and 3 only
B. 2 and 4 only
C. 1, 2 and 4 only
D. 1, 2, 3 and 4
Correct Option: C. 1, 2 and 4 only
1 March 2023 125

Who is the first Sahitya Akademi winner in


Bodo language?
A. Rita Basumatary
B. Mangal Singh Hazowary
C. Anil Boro
D. None of the above
Correct Option: B. Mangal Singh Hazowary
Mangal Singh Hazowary is an eminent Bodo
language poet. He is the recipient of
Sahitya Akademi Award for his poetry
"Ziuni Mwkthang Bisombi Arw Aroj" in 2005: the first
winner to win this award in Bodo language. He was
conferred the fourth highest civilian honour of
the Padma Shri, in 2021, for his contributions to
literature and education
1 March 2023 126

Consider the following statements regarding Ease


of Doing Business
1.It is published by IMF
2.India was ranked 75th out of 190 countries in EDB 2022
report.
Select the correct statements using the codes given below

A. 1 only
B. 2 only
C. Both
D. None of the above
Correct Option: D. None of the above
The The Ease of Doing Business (EoDB) index is a ranking system
established by the World Bank Group. India is ranked 63 among
190 economies in the ease of doing business, according to the
EDB 2022 report by World Bank.
1 March 2023 127

Which of the following is the nodal agency for the


implementation of Sustainable Development Goals in
India?
A. Ministry of Environment, Forest and Climate
Change
B. Ministry of Home Affairs
C. NITI Aayog
D. None of the above
Ans. C. NITI Aayog
NITI Aayog is the nodal agency in the Government
of India for overall coordination and supervision of
the Sustainable Development Goals.
1 March 2023 128

Which of the following organizations released the India


Skills Report 2022?

A. Ministry of Home Affairs


B. Ministry of Skill Development and Entrepreneurship
C. Wheebox Institution
D. NITI Aayog
Ans. C. Wheebox Institution
The 9th edition of India Skills Report (ISR)
2022, released by Wheebox, Maharashtra has
retained the top position in the list of states with the
highest poll of employable talent followed by Uttar
Pradesh and Kerala.
1 March 2023 129

Recently STUTI initiative is launched by

A. NITI Aayog
B. Ministry of Defence
C. Department of Science and Technology
D. Ministry of Finance
Ans. C. Department of Science and Technology
Synergistic Training program Utilizing the Scientific
and Technological infrastructure (STUTI) a new
initiative was recently unveiled by the Department
of Science and Technology.
1 March 2023 130

Regional Anti Terrorist Structure (RATS), seen in news, is


related to which of the following organization?

A. SCO
B. QUAD
C. BRICS
D. None of the above
Ans. A. SCO
• Regional Anti-Terrorism Structure is a permanent body
of the SCO with its headquarters in Tashkent.
• Its goal is to encourage member states to work
together in the fight against terrorism, separatism,
and extremism.
1 March 2023 131

Which of the following are classified as classical languages in India?


1. Tamil
2. Telugu
3. Odia
Select the correct option
A. 1 only
B. 1 and 2 only
C. 2 and 3 only
D. 1, 2 and 3
Ans. D. 1, 2 and 3
The Classical Languages in India are listed in the Eighth
Schedule of the Constitution.
There are six classical languages in India- Tamil,
Sanskrit, Kannada, Telugu, Malayalam and Odia
1 March 2023 132

The Aitchison Commission 1886 was related to?


A. Constitutional Reforms
B. Famine
C. Civil services
D. None of the Above

Ans. C. Civil Services


In 1886, Lord Dufferin established a "Public Service
Commission" headed by Sir Charles Aitchison to investigate the
problems of India's civil services. The Atchison Commission (Public
Service Commission) was established in 1886 under the chairmanship
of Sir Charles Umpherston Aitchison to devise a plan to address
Indians' claims to higher and more extensive employment in the public
sector.
1 March 2023 133

Logistics Performance Index report is released by?

A. World Bank
B. International Monetary Fund
C. World Economic Forum
D. None of the above
Ans. A. World Bank
Logistics Performance Index report is a bi-annual report
released by the World Bank
1 March 2023 134

Consider the following statements


1. The Treaty of Yandaboo was signed on the 24th of February, 1826
2. It ended the second Anglo-Burmese war.

Select the correct answer using the code given below

A. 1 Only
B. 2 Only
C. Both
D. Neither
Ans. A. 1 Only
Treaty of Yandaboo was the peace treaty that
resulted in end of First Anglo Burmese War. This
treaty was signed on February 24, 1826 after two
years of the war between British and Burmese
1 March 2023 135

Consider the following statements regarding UPI


1. It is a payment system developed by the NPCI
2. It is regulated by the Ministry of Finance
3. It is a system that powers multiple bank accounts into a single
mobile application, merging several banking features
Select the correct answer using the code given below

A. 1 and 2 Only
B. 2 and 3 Only
C. 1, 2 and 3 Only
D. 1 and 3 Only
Ans. D. 1 and 3 Only
UPI is a single window mobile payment system
developed by the NPCI. The interface is regulated by
the RBI. It is a system that powers multiple bank
accounts into a single mobile application, merging
several banking features.
1 March 2023 136

The NABH Nirman Scheme is related to

A. Providing Affordable Housing for Tribals


B. Civil Aviation Sector
C. Irrigation Sector
D. None of the above
Ans. B. Civil Aviation Sector
NABH Nirman initiative was first announced in Union
Budget of 2018.
❖ To expand the airport capacity more than five
times to handle a billion trips a year.
1 March 2023 137

Saubhagya Scheme is related to

A. Empowering rural ST women by providing loans


B. Providing electricity to ST & SC households
C. Providing electricity to households
D. Providing education loan to girls belongs to BPL
category
Ans. C. Providing electricity to households
Pradhan Mantri Sahaj Bijli Har Ghar Yojana
‘Saubhagya’ was launched in September, 2017.
Under Saubhagya free electricity connections to all
households (both APL and poor families) in rural areas and
poor families in urban areas will be provided.
1 March 2023 138

Consider the following statements regarding Monetary Policy


Committee
1. It is a 8 member body including the Governor of RBI.
2. It functions Under the Chairmanship of the Union Finance
Minister.
Select the incorrect statements using the codes given below
A. 1 Only
B. 2 Only
C. Both
D. Neither
Ans. C. Both
Under the RBI act, 1934, the central government is empowered to
constitute a six-member Monetary Policy Committee.
RBI Governor as its ex-officio chairperson.
1 March 2023 139

Consider the following statements (UPSC 2019)


1. Purchasing Power Parity (PPP) exchange rates are calculated by
comparing the prices of the same basket of goods and services in
different countries.
2. In terms of PPP dollars, India is the fifth largest economy in the
world.
Which of the statements given above is/are correct
A. 1 Only
B. 2 Only
C. Both 1 and 2
D. Neither 1 nor 2
Ans. A. 1 Only
On the PPP basis, China is the world’s largest economy in
2022, followed by the United States in 2nd and India in 3rd
position.
1 March 2023 140

Recently, PM-SHRI Scheme launched is related to

A. Education Development
B. Old Age pension
C. Women Empowerment
D. None of the Above

Ans. A. Education Development


14,500 schools across states and Union Territories will be redeveloped
to reflect the key features of the NEP, 2020 under the centrally
sponsored scheme known as PM SHRI Schools (PM Schools for Rising
India).
1 March 2023 141

Recently, Emission Gap Report 2022 is released by

A. UNDP
B. UNEP
C. WB
D. None of the above
Ans: B. UNEP
This is the 13th edition of the Report.
Title: ‘The Closing Window — Climate Crisis Calls For Rapid
Transformation of Societies’
1 March 2023 142

Q. At the state level, ministers are removed from office


A. By the state legislature after passing a resolution
with a majority of more than 50% of the total
membership of the state legislature.
B. By the chief minister.
C. By the governor on his discretion
D. By the governor on the aid and advise of the
council of ministers.

Ans. D. By the governor on the aid and advise of the


council of ministers.
1 March 2023 143

Recently seen in news, DMH-11 is a GM crop, variety of


which of the following?

A. Mustard
B. Soyabean
C. Brinjal
D. None of the above
Ans. A. Mustard

Dhara Mustard Hybrid – 11 or DMH-11 is a


genetically modified variety of mustard.
1 March 2023 144

International Labour Organization’s Conventions 138 and


182 are related to
A. Child Labour
B. Adaptation of agricultural practices to global climate
change
C. Regulation of food prices and food security
D. Gender parity at the workplace
Correct Option: A. Child Labour
The two International Labour Organization’s Conventions
Conventions on child labour are Convention No.138 on
Minimum Age and Convention No. 182 on the Worst Forms of
Child Labour. These Conventions are “fundamental”
Conventions. This means that, under the ILO Declaration on
Fundamental Principles and Rights at Work , all ILO member
States have an obligation to respect, promote and realize the
abolition of child labour , even if they have not ratified the
Conventions in question.
1 March 2023 145

e-Gram Swaraj portal has been launched by

A. Ministry of Panchayati Raj


B. Niti Aayog
C. Ministry of Rural Development
D. None of the above
Ans. A. Ministry of Panchayati Raj
The Ministry of Panchayati Raj launched e-Gram Swaraj
portal.
Aim: To bring in better transparency and strengthening of
the e-Governance in Panchayati Raj Institutions (PRIs)
across the country through decentralized planning,
progress reporting and work-based accounting.
1 March 2023 146

The Ninth Schedule was introduced in the Constitution


of India during the prime ministership of (UPSC 2019)

A. Jawaharlal Nehru
B. Lal Bahadur Shastri
C. Indira Gandhi
D. Morarji Desai
Ans. A. Jawaharlal Nehru
The Ninth Schedule became a part of the constitution,
when the constitution was amended for the first time
by the Nehru Government, on 10 May 1951.
1 March 2023 147

Which one of the following Indian City has won “World


Green Award 2022”

A. Guwahati
B. Delhi
C. Hyderabad
D. Chennai
Ans. C. Hyderabad
Hyderabad is the only Indian city that was selected for the
World Green City 2022.
The city has also won 'Living Green for Economic Recovery
and Inclusive Growth' at the International Association of
Horticulture Producers (AIPH) 2022 held in Jeju, South
Korea.
1 March 2023 148

Recently seen in news Miyawaki Method is related to


A. Water harvesting
B. Carbon footprint
C. Green accounting
D. Urban forestry

Ans. D. Urban forestry


Miyawaki is a technique pioneered by Japanese botanist
Akira Miyawaki, that helps build dense, native forests in
a short time. It has revolutionised the concept of urban
afforestation by turning backyards into mini-forests.
1 March 2023 149

Consider the following statements


1. During Swadeshi movement in Assam, Ambikagiri
Raychoudhury wrote a revolutionary drama “Bandini
Bharat”.
2. “Seva Sangha” a revolutionary organization was
established by Ambikagiri Raychoudhury.

Select the incorrect answer using the code given below


A. 1 Only
B. 2 Only
C. Both
D. Neither
Ans. D. Neither
1 March 2023 150

Who among the following is known as the “Steel


Man of India”?

A. Dadabhai Naoroji
B. Vallabhbhai Patel
C. Jamshed J Irani
D. Lal Bahadur Shastri

Ans. C. Jamshed J Irani


Recently, Jamshed J Irani, also known as the
"Steel Man of India", passed away aged 86 at
Tata Main Hospital in Jamshedpur.
1 March 2023 151

Consider the following statements regarding Rahmat Banu Begum


1. She is also known as Nangchen Gabharu.
2. She was the first wife of Mughal emperor Muhammad Azam
Shah.
3. She was sent to the Mughal harem as part of the Treaty of
Ghilajharighat.

Select the correct statements using the code given below

A. 1 and 2 only
B. 1 and 3 only
C. 2 and 3 Only
D. 1, 2 and 3

Ans. D. 1, 2 and 3
1 March 2023 152

The treaty of Badarpur was singed between David


Scott and Govinda Chandra in which year?

A. 1824
B. 1826
C. 1832
D. 1835

Ans. A. 1824
1 March 2023 153

From whose regin Sanskrit language used on Ahom


coins

A. Gadadhar Singha
B. Shiv Singha
C. Rudra Singha
D. None of the above

Ans. C. Rudra Singha


1 March 2023 154

HARIT Aaykar is an initiative of

A. Central Board of Direct Taxes


B. Reserve Bank of India
C. National Highways Authority of India
D. None of the Above
Ans. A Central Board of Direct Taxes
The Income Tax Department has launched the HARIT
Aaykar (Hariyali Achievement Resolution by Income
Tax) Initiative on the occasion of National Unity Day.
1 March 2023 155

Wangala Dance is a Festival Dance of which North East


Indian Tribe?

A. Garo
B. Khasi
C. Tangsa
D. Kuki
Ans. A. Garo
Wangala is also called the festival of “The Hundred Drums“.
It is a harvest festival celebrated by the Garo tribe in
Meghalaya, Nagaland and Assam and Greater Mymensingh
in Bangladesh.
1 March 2023 156

Consider the following statements regarding IPCC


1. IPCC is a joint initiative of World Metrological Organizations and
UNDP.
2. IPCC was awarded Nobel Peace Prize in 2007.
Select the correct statements using the codes given below
A. 1 only
B. 2 only
C. Both
D. Neither
Ans. B. 2 Only
The Intergovernmental Panel on Climate Change (IPCC) is the UN
body for assessing the science related to climate change.
Established by the United Nations Environment Programme (UNEP)
and the World Meteorological Organization (WMO) in 1988.
IPCC was awarded Nobel Peace Prize in 2007.
1 March 2023 157

Which of the following pairs of dances and states is


incorrect

A. Bhortal Dance- Assam


B. Fugdi- Maharashtra
C. Popir - Arunachal Pradesh
D. Pung Cholom- Manipur

Ans. Option B is incorrect


Fugdi Dance - Goa

Fugdi Dance - Goa


1 March 2023 158

Garuda is a bilateral exercise between


A. India and Bangladesh
B. India and France
C. India and Indonesia
D. India an Nepal

Ans. B. India and France


It is a bilateral exercise between Indian Air Force (IAF)
and French Air and Space Force (FASF).
1 March 2023 159

Consider the following statements


1. During the second Burmese invasion of Assam,
Burmese reinstalled Purandar Singha as a king.
2. Sudangphaa was popularly known as Bamuni Konwar.
Select the correct statements using the codes given
below

A. 1 only
B. 2 only
C. Both 1 and 2
D. Neither 1 nor 2

Ans. B. 2 Only
1 March 2023 160

Ranthambore National Park is situated in


A. Rajasthan
B. Madhya Pradesh
C. Gujarat
D. Chhattisgarh

Ans. A. Rajasthan

Ranthambore National Park is located in Rajasthan. It is located at


the junction of the Aravali and Vindhya hill ranges. It is the natural
habitat to the significant number of Royal Bengal tigers.
Ranthambore National Park was established initially as Sawai
Madhopur Game Sanctuary in 1955 by the Government of India.
In 1973, it was declared as one of the Project Tiger reserves in
India.
1 March 2023 161

Recently, in the news seen, the word 'Denisovan' in


reference to

A. fossils of a kind of dinosaurs


B. an early human species
C. a cave system found in North-East India
D. a geological period in the history of Indian
subcontinent
Ans. B. an early human species
1 March 2023 162

Recently seen in the news, the ‘Bru agreement’ has been signed
between?

A. Nagaland and Mizoram


B. Assam and Meghalaya
C. Mizoram and Tripura
D. Manipur and Assam
Ans. C. Mizoram and Tripura

The Bru-Reang agreement is signed in New Delhi by


representatives from the Indian government, led by the Home
Minister for Home Affairs, the governments of Tripura and
Mizoram, and Bru-Reang.
1 March 2023 163

Which is NOT correct with respect to which countries are


participating in the Moscow Format of Consultations on
Afghanistan ?
1) Russia, India, China, Pakistan, and Iran
2) Turkmenistan, Kazakhstan, Uzbekistan, Kyrgyzstan and
Tajikistan
3) Russia, Indian, China, Pakistan, Iran, and Iraq
4) Turkmenistan, Kazakhstan, Azerbaijan, Kyrgyzstan and
Tajikistan
A. 1 and 2 only
B. 3 and 4 only
C. 1 and 4 only The countries participation in the
D. 2 and 3 only talks include Russia, India, China,
Pakistan, Iran, Turkmenistan,
Kazakhstan, Uzbekistan,
Ans. B. 3 and 4 only Kyrgyzstan and Tajikistan
1 March 2023 164

Consider the following statements regarding to the


Citadel of Indus Valley Civilization
1. It was called the lower town.
2. It was present on the eastern side and was larger as
compared to the other part.
Select the correct answer using the codes given below

A. 1 Only
B. 2 Only
C. Both 1 and 2
D. Neither 1 nor 2
Ans. D. Neither 1 nor 2
1 March 2023 165

Statements:
1.In the Allahabad Pillar Inscription of Samudragupta, the
king of Kamarupa is mentioned as Pratyanta Nripati
2.The Shung-Shu (a Chinese account) refers to
the sending of an embassy to China in 428 A.D. by an
Indian king called Yu-Chai, which as per historian,
was Kalyana Varman
Which of the above statements are incorrect?
A. Only 1
B. Only 2
C. Both
D. None of the above
Correct Option: D. None of the above
1 March 2023 166

Consider the following statements regarding to the


Regulating Act of 1773
1. It established Executive council of four members to
assist the governor general of Bengal.
2. Board of Control was established to manage political
affairs.
3. It ended the system of double government.

Select the incorrect statements using the codes below

A. 1 and 2 only
B. 2 and 3 only
C. 1 only
D. 3 only
Ans. B. 2 and 3 Only
1 March 2023 167

Consider the following statements regarding to the


government of India Act, 1919

1. Dyarchy was introduced in provinces.


2. Provincial Legislative councils were expanded
3. Voting rights was further curtailed.

Select the correct statements using the codes below

A. 1 only
B. 1 and 2 only
C. 1 and 3 only
D. 2 and 3 only

Ans. B. 1 and 2 Only


1 March 2023 168

Consider the following statements regarding Continental


Drift Theory
1. It was given by Alfred Wegener in 1912.
2. All the continents formed a single continental mass
called Pangea.
3. There existed a mega ocean called Panthalassa that
surrounded Pangea.
Select the correct statements using the code given below

A. 1 only
B. 1 and 2 only
C. 2 and 3 only
D. 1, 2 and 3
Ans. D. 1, 2 and 3
1 March 2023 169

Consider the following statements regarding Government of


India Act of 1858
1. The British Governor-General of India was given the title of
Viceroy.
2. It established 20 member council which was an advisory
body.
3. It abolished the board of control and court of directors.
4. It established new legislative council for Bengal.

Select the incorrect statements using the codes given below

A. 1 and 3 Only
B. 2 and 3 Only
C. 1 and 4 Only
D. 2 and 4 Only

Ans. D. 2 and 4 Only


1 March 2023 170

Consider the following statements regarding Mohenjo-Daro


1. It is situated on the bank of river Ravi.
2. Archaeological ruins at Mohenjo-Daro, were discovered in
1922 by Rakhaldas Banerji.
Select the incorrect statements using the codes given below
A. 1 only
B. 2 only
C. Both 1 and 2 It is situated on the bank of river
Indus in Sindh province.
D. Neither 1 nor 2
Archaeological ruins at Mohenjo-
Ans. A. 1 Only Daro, were discovered in 1922 by
Mohenjo-Daro, which means Rakhaldas Banerji.
mound of the dead was one of the Major Findings- dancing girl statue,
main centres of ancient Indus Pashupati seal, Statue of bearded
Valley Civilization. man etc.
1 March 2023 171

Arittapatti Biodiversity Heritage site is located in

A. Kerala
B. Andhra Pradesh
C. Tamil Nadu
D. Karnataka

Ans. C. Tamil Nadu


Recently, the Tamil Nadu Government issued a
notification declaring Arittapatti Biodiversity Heritage site
the first biodiversity heritage site in the state.
Nallur Tamarind Grove in Bengaluru Karnataka was the
first Biodiversity Heritage Site of India, declared in 2007
1 March 2023 172

Consider the following statements


1. Communal award restricted separate electorates for the
Muslims, Sikhs, Indian Christians, Anglo Indians and Europeans
only.
2. Indian councils act of 1909, provided for the association of
Indians with executive councils of the Viceroy only.

Select the correct statements using the codes given below

A. 1 Only
B. 2 Only
C. Both 1 and 2
D. Neither 1 nor 2
Ans. D. Neither 1 nor 2
1 March 2023 173

Consider the following statements regarding to the Government of India


Act of 1919
1. It introduced, for the first time, bicameralism and direct elections in the
country.
2. It provided for the separate representation of presidency corporation,
chambers of commerce, university and zamindars.
3. It separated, for the first time, provincial budgets from the Central
budget and authorised the provincial legislatures to enact their budgets.

Select the correct statements using the codes given below

A. 1 and 2 only • It separated, for the first time,


provincial budgets from the
B. 1 and 3 only
Central budget and authorised the
C. 2 and 3 only provincial legislatures to enact
D. 1, 2 and 3 their budgets. So, statement 1
and 3 are correct
Ans. B. 1 and 3 Only
Statement 2 is incorrect: Indian
Government of India Act 1919: Councils act 1909, provided for the
• It introduced, for the first time,separate representation of
bicameralism and direct electionspresidency corporation, chambers of
in the country. commerce, university and zamindars
1 March 2023 174

Consider the following statements regarding Suklenmung


1. He Shifted the capital of Ahom from Charguya to
Gargaon.
2. Garhgaon Tank was excavated during his reign.

Select the correct statements using the codes given


below

A. 1 Only Suklenmung
He shifted the capital to Gargaon
B. 2 Only from Bokota, hence called
C. Both 1 and 2 Gargainya Raja.

D. Neither 1 nor 2 Under his supervision, Gargaon


Pukhuri was dug, and Naga Ali
Ans. B. 2 Only (Gadhuli Bazar Mauza to Naga
Ali) was constructed.
1 March 2023 175

Match the following w.r.t. bilateral army drills of India


1. Yudh Abhyas - USA
2. Agni Warrior - Singapore
3. Harimau Shakti - Malaysia
4. Garuda Shakti - Indonesia
How many pairs given above are correctly matched ?

A. Only one pair


B. Only two pairs
C. Only three pairs
D. All four pairs
Ans. D. All four pairs
1 March 2023 176

Consider the following statements


1. Deflation refers to the decrease in the general price level
of goods and services but inflation rate remains above
zero.
2. Stagflation is a situation in which the inflation rate is low
and at the same time economic growth rate remains
steadily high.
Select the incorrect statements using the codes given below

A. 1 Only
B. 2 Only
C. Both 1 and 2
D. Neither 1 nor 2
Ans. C. Both 1 and 2
1 March 2023 177

Consider the following statements regarding SVAMITVA


scheme
1. It is a Central Sector Scheme
2. The scheme is piloted by the Ministry of Panchayati Raj.
3. Under the scheme, residential land in villages will be
measured using drones to create a non-disputable
record
Select the incorrect statements using the codes given below
A. 1 and 2 Only Villages and Mapping with
B. 2 and 3 Only Improvised Technology in Village
Areas.
C. 3 Only
It is a Central Sector Scheme which
D. None of the above was launched nationally on the
Ans. D. None of the above occasion of National Panchayati Raj
Day on 24th April 2021.
SVAMITVA stands for Survey of
1 March 2023 178

Consider the following statements


1. Real GDP is the GDP derived after adding the effect
of Inflation.
2. The difference between the real and nominal GDP
shows the levels of inflation in the year.
Select the incorrect statements using the codes given
below
A. 1 Only the value of all the goods
B. 2 Only and services used to
calculate the GDP of the
C. Both 1 and 2 country at the base year
D. Neither 1 nor 2 prices. In others words, Real
GDP is the GDP of the
Ans. A. 1 Only country adjusted to
Real GDP, can be defined as inflation.
1 March 2023 179

Consider the following statements regarding David Scott


1) The foundations of British administration in Assam were laid
by David Scott
2) David Scott abolished the existing Paik and Khel system in
Upper Assam
Select the incorrect statements using the codes given below
A. 1 only
B. 2 only
C. Both 1 and 2
D. Neither 1 nor 2

Ans. B. 2 Only
In Upper Assam, David Scott retained the existing Paik and Khel
system with some minor changes in the revenue system.
1 March 2023 180

Consider the following statements regarding Kaziranga National Park


(KNP)
1. Recently, Several measures including anti-poaching camps,
alternate livelihood training were undertaken at KNP as part of
Indo- pacific Parks Partnership.
2. KNP has the largest-population of one-horned rhinoceros in the
world.
Select the correct statements using the codes given below
A. 1 only livelihood training were undertaken
B. 2 only at KNP as part of Indo- pacific Parks
Partnership.
C. Both 1 and 2 KNP is a UNESCO World Heritage
D. Neither 1 nor 2 site and has the largest-population
Ans. C. Both 1 and 2 of one-horned rhinoceros in the
world.
Recently, Several measures
including artificial highlands where
animals can escape during floods;
anti-poaching camps, alternate
1 March 2023 181

Consider the following statements regarding India Inequality Report 2022:


Digital Divide
1. It is an annual report to highlight ongoing inequality cries in the
country.
2. Gujarat has highest internet penetration, while Bihar has the lowest.
3. Use of computer devices has increased post covid period in rural
areas.
Select the incorrect statements using the codes given below
A. 1 and 2 only Oxfam India is a non-profit
B. 2 and 3 only organization working to support
child education, empowering
C. 1 and 3 only women & fighting against
D. 3 only inequality in India. Maharashtra
has highest internet penetration,
Ans. B. 2 and 3 only while Bihar has the lowest.
It is an annual report to highlight Use of computer devices has
ongoing inequality cries in the decreased post covid period in
country released by Oxfam India. rural areas
1 March 2023 182

Consider the following statements regarding Monkeypox


1. Monkeypox is a zoonosis, that is a disease that is
transmitted from infected humans to humans.
2. The first case of monkeypox was reported in 1958 in
monkeys.
Select the correct statements using the codes given below
A. 1 only humans in 1970 in the western
B. 2 only Africa.
C. Both 1 and 2 Recently, WHO has declared the
global monkey monkeypox
D. Neither 1 nor 2 outbreak a “Public health
emergency of international
Ans. B. 2 Only concern” means an extraordinary
Monkeypox is a zoonosis, that is aevent.
disease that is transmitted from
infected animals to humans. The
first case of monkeypox was
reported in 1958 in monkeys and in
1 March 2023 183

Consider the following statements Archaeological Survey of India (ASI)


1. It is an agency attached to the Ministry of Culture.
2. It was founded in 1861 by Alexander Cunningham.
Select the correct statements using the codes given below
A. 1 only became its first Director –General.
B. 2 only Alexander Cunningham is also known
C. Both 1 and 2 as the Father of Indian Archaeology.
ASI has also undertaken major
D. Neither 1 nor 2
conservation works abroad besides
Ans. C. Both 1 and 2 carrying out excavations,
ASI, under the Ministry of Culture, is explorations, images and other
the premier organization for the studies in countries like Afghanistan,
archaeological researches and Nepal, Cambodia and Egypt.
protection of the cultural heritage of
India. It was established in 1861 by
Alexander Cunningham who also
1 March 2023 184

Which among the following events happened earliest?

A. Swami Dayanand established Arya Samaj


B. Dinabandhu Mitra wrote Neeldarpan
C. Bankim Chandra Chattopadhyay wrote Anandmath
D. Satyendranath Tagore became the first Indian to succeed in
the Indian Civil Services Examination
Ans. B. Dinabandhu Mitra wrote Neeldarpan
1859: Neeldarpan written by Dinabandhu Mitra
1863: Satyendranath Tagore became the first Indian to succeed
in the Indian Civil Services Examination
1875: Swami Dayanand established Arya Samaj
1882: Bankim Chandra Chattopadhyay wrote Anandmath
1 March 2023 185

Consider the following statements.


1. Tax Buoyancy refers to the responsiveness of tax revenue
growth to changes in a country’s Gross Domestic Product (GDP).
2. Greenwashing is the practice of conveying a false impression
that a company’s products are eco-friendly and environmentally
sound.
Select the correct statements using the codes given below
A. 1 only
B. 2 only revenue growth to changes in GDP.
C. Both 1 and 2 The act of giving the false
D. Neither 1 nor 2 impression that a company’s
products are ecologically friendly is
Ans. C. Both 1 and 2 known as “greenwashing.”
Tax buoyancy is an Economic The term greenwashing was first
theory, explains the relationship used in 1986 by Jay Westerveld, an
between the changes in the American environmentalist and
government’s tax revenue growth researcher.
and the changes in GDP. It refers
to the responsiveness of tax
1 March 2023 186

Consider the following statements regarding OECD


1. It is an international forum headquartered at Paris.
2. India is a member of the OECD.
3. OECD’s Development Assistance Committee is a 31 member group
which works to promote development co-operation and 2030 Agenda
for Sustainable Development
Select the correct statements using the codes given below
A. 1 and 2 Only India is not a member of OECD but
one of those non-member economies
B. 2 and 3 Only
with which the OECD has working
C. 1 and 3 Only relationships in addition to its
D. 3 only member countries.

Ans. C. 1 and 3 Only OECD’s Development Assistance


Committee (DAC) is a 31 member
OECD is an international forum group which works to promote
headquartered at Paris. development co-operation and 2030
OECD has 38 member countries. Agenda for Sustainable Development
1 March 2023 187

Consider the following statements


1. Devicharan Baruah attended the second session of the Congress
at Calcutta.
2. Lokpriya Gopinath Bordoloi also attended the second session of
the Congress.
3. Manik Chandra Barooah attended the sixth session of the
Congress at Calcutta.
Select the correct statements using the codes given below
A. 1 and 2 Only Bordoloi (not Bharat Ratna
B. 2 and 3 Only Gopinath Bordoloi) attended the
second session of the Congress at
C. 1 and 3 Only Calcutta.
D. 1 Only Manik Chandra Barooah attended
Ans. C. 1 and 3 Only the sixth session of the Congress at
Calcutta
Devicharan Baruah and Gopinath
1 March 2023 188

Consider the following statements regarding to Gopal Hari


Deshmukh
1. He was the editor of the Lokhitwadi, a monthly magazine in
Marathi
2. Pamphlets like ‘Nibandha Sangraha’, ‘Vidyalahari’ were credited
to his name
Select the correct statements using the codes given below
A. 1 only arresting the progress of the
Hindu society by fostering
B. 2 only
antisocial traditions.
C. Both 1 and 2
Pamphlets like ‘Nibandha
D. Neither 1 nor 2 Sangraha’, ‘Vidyalahari’,
Ans. C. Both 1 and 2 ‘Hindustanalit Bal Vivah’, ‘Agam
Prakash’ , and ‘Nigam Prakash’
Gopal Hari Deshmukh was born in were credited to his name. He
Poona in 1823. He edited the wrote a series of letters under the
Lokhitwadi, a monthly magazine pen name Lokhitwadi in a weekly
in Marathi. Through his writings newspaper Prabhakar.
he criticized the Brahmins for
1 March 2023 189

Consider the following statements

1. According to Laffer curve, tax revenues always increase with


increase in tax rate.

2. Lorenz curve is a graph showing the degree of inequality in income


in a given population or an economy.

Select the correct statements using the codes given below

A. 1 only The Laffer curve states that if tax


B. 2 only rates are increased above a
certain level, then tax revenues
C. Both 1 and 2 can actually fall because higher
D. Neither 1 nor 2 tax rates discourage people from
Ans. B. 2 Only working. The curve show the
relationship between tax rates and
Lorenz curve is a graphical the amount of tax revenue
representation of the distribution collected by governments.
of income or of wealth.
1 March 2023 190

Match the List-I with List-II


List-1 List-2
A. Emergency Provisions 1) Ireland

B. Fundamental Rights 2) UK
C. Parliamentary System 3) USA

D. DPSP 4) Germany
Select the correct answer using the cods given below
A. A-4, B-1, C-2, D-3
Emergency Germany
B. A-2, B-1, C-4, D-3 Provisions
C. A-2, B-3, C-4, D-1 Fundamental USA
Rights
D. A-4, B-3, C-2, D-1
Parliamentary UK
Ans. D. A-4, B-3, C-2, D-1 System
DPSP Ireland
1 March 2023 191

Consider the following statements regarding GST


1. The decisions of the GST Council are binding on Parliament.
2. GST rates are notified on the recommendations of the Finance
ministry.
Select the incorrect statements using the codes given below
A. 1 Only simultaneously levying tax on a
B. 2 Only common base. GST rates are
notified on the recommendations
C. Both 1 and 2 of the GST Council.
D. Neither 1 nor 2 The decisions of the council are
Ans. B. 2 Only binding on Parliament.
The Goods and Services Tax (GST) is
a value added tax levied on most
goods and services sold for
domestic consumption. It is a dual
GST with Centre and states
1 March 2023 192

Consider the following statements regarding Indian Knowledge


System (IKS)
1. It is an innovative cell under the Ministry of Education.
2. It was established in October 2020.
Select the correct statements using the codes given below
A. 1 only It was established in October
B. 2 only 2020.
C. Both 1 and 2 It was established to promote
interdisciplinary research on all
D. Neither 1 nor 2 aspects of IKS, preserve and
Ans. C. Both 1 and 2 disseminate Indian Knowledge
System for further research and
Indian Knowledge System (IKS) actively engage for spreading
is an innovative cell under the the rich heritage and traditional
Ministry of Education (MoE) at knowledge of our country.
AICTE.
1 March 2023 193

Which one of the following sites is famous for


prehistoric paintings?

A. Bhimbetka
B. Ajanta
C. Bagh
D. None of the above
Ans. A. Bhimbetka
The Rock shelters of Bhimbetka are a repository of rock
paintings within natural rock shelters with archaeological
evidences from the Palaeolithic and Mesolithic periods through
the Chalcolithic to the Medieval Period.
1 March 2023 194

Consider the following statements regarding District Planning Committee


1. Head of Panchayat samiti functions as Secretary to the District
Planning Committee.
2. The Committee in each district should consolidate the plans prepared
by the Panchayats and the Municipalities in the district
Select the incorrect statements using the codes given below

A. 1 Only Secretary to the District Planning


Committee.
B. 2 Only
C. Both 1 and 2 The Committee in each district should
consolidate the plans prepared by the
D. Neither 1 nor 2 Panchayats and the Municipalities in
Ans. A. 1 Only the district

District Planning Committee created It also prepares a draft development


as per article 243ZD of the plan for the district.
constitution of India
District Collector functions as
1 March 2023 195

Consider the following statements


1. Venus has retrograde rotation.
2. Neptune has the largest diurnal range of temperature.
3. Mercury has one of the lowest temperature.
Select the incorrect statements using the codes given below

A. 1 and 2 Only range of temperature, day and


B. 1 and 3 Only night temperatures at Mercury
are the 430 degree Celsius and -
C. 2 and 3 Only 180 degree Celsius
D. 3 Only Neptune has one of the lowest
temperature of -200
Ans. C. 2 and 3 Only degree Celsius
Uranus and Venus are two planets
of the solar system which have
retrograde rotation.
Mercury has the largest diurnal
1 March 2023 196

Consider the following pairs


Protected area State
1. Barnadi wildlife sanctuary Assam
2. Pakhui tiger reserve West Bengal
3. Netravali wildlife sanctuary Odisha

Select the correct pairs using the cods given below


A. 1 and 3 Only
B. 2 and 3 Only Pakke Tiger Reserve -located in the
C. 1 Only Pakke Kessang district of
Arunachal Pradesh.
D. 3 Only
Ans. C. 1 Only Netravali wildlife sanctuary is
situated in the state of Goa.
Barnadi Wildlife sanctuary- located
in northern Assam’s Baksa and
Udalguri districts bordering Bhutan.
Pakhui tiger reserve- also known as
1 March 2023 197

Consider the following statements regarding Treaty of Salbai


1. It was signed between the Marathas and the British East India
Company.
2. It was the outcome of the second Anglo Maratha War.
Select the correct statements using the codes given below
A. 1 Only
B. 2 Only
C. Both 1 and 2
D. Neither 1 nor 2
Ans. A. 1 Only
Treaty of Salbai (17 May 1782) was signed between the Marathas and
the British East India company. It was the outcome of the First Anglo
Maratha War. As per the treaty, British acknowledged Madhavrao II
Narayan as Peshwa of the Maratha Empire.
1 March 2023 198

Consider the following pairs


Harappan site Feature
1. Balu Garlic
2. Allahdino Jewellery
3. Dholavira Signboard
Select the correct pairs using the cods given below
A. 1 and 2 only of Garlic has been found.
B. 2 and 3 only Allahdino - near Karachi, a lot of
jewellery made up of gold, silver,
C. 1 and 3 only copper, carnelian, and other
D. 1, 2 and 3 precious stones have been
Ans. D. 1, 2 and 3 unearthed
Balu is a Harappan site located in Dholavira - a large wooden
the state of Haryana from where "signboard" just outside the
the evidence for the production north entrance to the citadel.
1 March 2023 199

Consider the following statements regarding the Harappan Civilization


1. The Great Bath was housed in the lower town and was used for
ritual bathing.
2. Presence of metallic money encouraged large scale trade.
Select the correct statements using the codes given below
A. 1 Only
B. 2 Only
C. Both 1 and 2
D. Neither 1 nor 2
Ans. D. Neither 1 nor 2
The Great Bath is one of the important structure located in the Citadel. It was a
structure of public importance and was used for ritual bathing. It was
rectangular in shape surrounded by the corridors on the three sides and had
rooms for changing as well.
The trade during the Harappan times was largely based on the barter system as
there was no metallic money in use.
1 March 2023 200

Consider the following statements regarding the Global Gender Gap Index
1. It is Published by the World Economic Forum.
2. India ranks 135 among a total of 146 countries in the Global Gender Gap
Index 2022
Select the correct statements using the codes given below

A. 1 Only
B. 2 Only Key dimensions–
C. Both 1 and 2 ▪ Economic participation and
opportunity
D. Neither 1 nor 2
▪ Educational attainment
Ans. C. Both 1 and 2 ▪ Health and survival
The Global Gender Gap index ▪ Political empowerment
designed to measure gender equality
which is published by the World
Economic Forum. India ranks 135
among a total of 146 countries in the
index.
1 March 2023 201

Consider the following statements regarding to the National commission for


backward classes
1. It is a non- constitutional body.
2. The conditions of service and tenure of the chairperson is determined by
the President.
3. It is a statutory body under the Ministry of social justice and
Empowerment.
Select the incorrect statements using the codes given below
A. 1 and 2 Only the National Commission for Backward
Classes (NCBC) . The conditions of
B. 1 and 3 Only
service and tenure of office of the
C. 2 and 3 Only Chairperson, Vice-Chairperson and
other Members is determined by the
D. 2 Only
President. The Commission consists of
Ans. B. 1 and 3 Only five members including a Chairperson,
Vice-Chairperson and three other
Previously NCBC was a statutory body
Members appointed by the President
under the Ministry of Social Justice and
by warrant under his hand and seal.
Empowerment.
102nd Constitution Amendment Act,
2018 provides constitutional status to
1 March 2023 202

Consider the following statements regarding to the Governor


1. The Governor can only appoint members of the state Public Service
Commission, he cannot remove them.
2. Governor can also pardon the death sentence of a person.
3. The Same person can be appointed as the Governor for two or more states.
4. The oath of office to the Governor is administered by the President of India.
Select the incorrect statements using the codes given below
A. 1, 2 and 3 Only appointment of the same person as a
governor for two or more states.
B. 1 and 4 Only
The oath of office to the Governor is
C. 2 and 4 Only administered by the Chief justice of the
D. 3 and 4 Only concerned state high court and in his
absence, the senior-most judge of that
Ans. C. 2 and 4 Only court available
The Governor appoints the chairman
and members of the State public service He cannot pardon a death sentence.
commission. However, they can be Even if a state law prescribes for the
removed only by the president. death sentence, the power to grant
pardon lies with the President and not
Usually, there is a governor for each with the governor.
state, but the 7th Constitutional
Amendment Act of 1956 facilitated the
1 March 2023 203

Consider the following pairs


Cause of withdrawal
Movement
1. Non- Cooperation Chauri Chaura incident
Movement
2. CDM Poon Pact

Select the correct pairs using the cods given below


A. 1 Only Gorakhpur, Uttar Pradesh.
B. 2 Only Gandhi immediately withdrew the
movement as incidents of violence
C. Both 1 and 2 would give an excuse
D. Neither 1 nor 2 to the Government for the brutal
actions.
Ans. A 1 Only
The Gandhi-Irwin Pact of 1931 was
The immediate reason for the the reason for the withdrawal of
withdrawal of the Non-cooperation the Civil Disobedience movement.
Movement was the eruption of
violence at Chauri Chaura,
1 March 2023 204

Consider the following statements


1. The Chief justice of the concerned state high court may be appointed
temporarily to discharge the functions of the governor of that state.
2. The governor can be removed from office on the ground of the violation of
the Constitution as stated in the Constitution.
Select the incorrect statements using the codes given below
A. 1 Only temporarily as the governor of that
state.
B. 2 Only
The Constitution does not lay down any
C. Both 1 and 2 grounds upon which a governor may be
D. Neither 1 nor 2 removed by the President. The
President may transfer a Governor
Ans. B. 2 Only appointed to one state to another state
The President may make such provision for the rest of the term. Further, a
as he thinks fit for the discharge of the Governor whose term has expired may
functions of the governor in any be reappointed in the same state or any
contingency not provided in the other state.
Constitution, for example, the death of
a sitting governor. Thus, the chief justice
of the concerned state can be appointed
1 March 2023 205

Consider the following statements


1. The Constitution of India does not contain any specific procedure for
the selection and appointment of the Prime Minister
2. The term of the office of the Prime Minister is fixed for 5 years
3. He/ She does not have the power to recommend the dissolution of the
Lok Sabha to the President.
Select the incorrect statements using the codes given below
A. 1 and 2 Only The term of the Prime Minister is not
fixed and he holds office during the
B. 1 and 3 Only
pleasure of the president. However,
C. 2 and 3 Only this does not mean that the president
can dismiss the Prime Minister at any
D. 3 Only
time.
Ans. C. 2 and 3 Only
The Prime Minister is the leader of
The Constitution does not contain any the Lower House. In this capacity, He
specific procedure for the selection / She can recommend dissolution of
and appointment of the Prim the Lok Sabha to President at any
Minister. Article 75 says only that the time.
Prime Minister shall be appointed by
the president.
1 March 2023 206

Consider the following statements regarding Sixth Schedule


1. Sixth schedule extends to state of Assam, Meghalaya, Mizoram and
Manipur.
2. President can reorganize and alter the boundaries of autonomous
districts under the schedule.
Select the incorrect statements using the codes given below

A. 1 only states of Assam, Meghalaya, Tripura


and Mizoram.
B. 2 only
C. Both 1 and 2
The governor is empowered to
D. Neither 1 nor 2 organize and re-organize the
Ans. C. Both 1 and 2 autonomous districts. Thus, he can
increase or decrease their areas or
The Constitution, under Sixth change their names or define their
Schedule, contains special provisions boundaries and so on.
for the administration of tribal areas
in the four north-eastern
1 March 2023 207

Consider the following statements regarding National Commission for


Scheduled Castes (NCSC)
1. It consists of a chairperson and three other members.
2. NCSC has to investigate all matters relating to the constitutional and
other legal safeguards for the Anglo-Indian community
Select the incorrect statements using the codes given below

A. 1 Only
B. 2 Only According to Article 338
C. Both 1 and 2 (10) of the Constitution, the
D. Neither 1 nor 2
Commission has to investigate all
matters relating to the
Ans. A. 1 Only constitutional and other legal
NCSC consists of a chairperson, a safeguards for the Anglo-Indian
vice-chairperson and three other Community and report to the
members. President upon their working.
1 March 2023 208

Consider the following statements regarding Special Officer for Linguistic


Minorities
1. The provision for the Special Officer for Linguistic Minorities has been made
under Part XVI of the Constitution.
2. The Constitution provides that the Special Officer reports to the President
annually relating to the safeguards provided for linguistic minorities under
the constitution.
3. The Special Officer is appointed by the President of India.
Select the incorrect statements using the codes given below
A. 1 Only
B. 1 and 2 Only Article 350B(2) provides that the
C. 2 and 3 Only Special Officer shall report to the
President upon those matters at such
D. 1 and 3 Only intervals as the President may direct.
Ans. B. 1 and 2 Only There is no provision for annual
reports.
The provision for the Special Officer
for Linguistic Minorities has been Article 350B(1) provides that the
made under Article 350B of the Special Officer to be appointed by the
Constitution which is in Part XVII President
(Official language) of the
Constitution.
1 March 2023 209

Consider the following statements regarding Ahmedabad session of the


Congress
1. Poorna Swaraj was demand in the Ahmedabad Session, 1921 by C.R
Das.
2. C. R Das presided over the Ahmedabad session in the absence of the
elected President
Select the correct statements using the codes given below
A. 1 Only
B. 2 Only
C. Both 1 and 2
D. Neither 1 nor 2
Ans. D. Neither 1 nor 2
In the absence of C. R. Das, who was in prison, Hakim Ajmal Khan
acted as the president of the Congress in the Ahmedabad session,
1921. It was in this session that Hazrat Mohani demanded Poorna
Swaraj.
1 March 2023 210

Consider the following pairs


Books/Newspaper Authors
1. Father of Indian Unrest Bal Gangadhar
2. Bande Mataram Bipin Chandra Pal
3. Leader Madan Mohan Malaviya

Select the correct pairs using the cods given below

A. 1 only Chirol.
B. 1 and 3 only Bande Mataram was started by
C. 2 and 3 only Bipin Chandra Pal in 1905.
D. 2 only The Leader newspaper was
Ans. C. 2 and 3 Only started by Madan Mohan
Malaviya in English.
The Father of Indian Unrest was
based on Bal Gangadhar Tilak.
However, it was not written by
him. It was written by Valentine
1 March 2023 211

Consider the following Pairs


Schedule Provision
1. Fourth Schedule Allocation of seats in the Lok Sabha to
the states
2. Eight Schedule Languages
3. Ninth Schedule Disqualification of the members of
Parliament
Select the incorrect pairs using the cods given below
A. 1 and 2 Only Constitution. There are 22 official
languages recognized by the Indian
B. 2 and 3 Only Constitution.
C. 1 and 3 Only Ninth schedule deals with the
D. 3 Only state acts and regulations that deal
with land reforms and abolition of
Ans. C. 1 and 3 Only the zamindari system. It also deals
with the acts and regulations of the
Fourth Schedule Deals with the Parliament dealing with other
Allocation of seats in the Rajya matters.
Sabha (the Upper House of
Parliament) to the State and Union
Territory.
Eight schedule deals with the
Languages recognized in the
1 March 2023 212

Consider the following statements regarding Federal Court in India


1. It was established as per the provisions of the Regulating Act of
1773
2. Sir Elijah Impey was the first Chief Justice of this court.
Select the correct statements using the codes given below
A. 1 Only
B. 2 Only
C. Both 1 and 2
D. Neither 1 nor 2
Ans. D. Neither 1 nor 2
The Federal Court was created by the Government of India Act, 1935.
Sir Maurice Gwyer was the first chief Justice of the Federal Court.
Sir Elijah Impey was the first chief justice of the Supreme Court
established under the Regulating Act of 1773
1 March 2023 213

Consider the following statements regarding Ramsar Convention


1. It is the only international treaty that addresses a specific
ecosystem.
2. It is an intergovernmental treaty that provides the
framework for the conservation and wise use of wetlands
and their resources.
Select the correct statements using the codes given below
A. 1 Only conservation and wise use of
B. 2 Only wetlands and their resources.
The Convention was adopted in
C. Both 1 and 2 the Iranian city of Ramsar in 1971
D. Neither 1 nor 2 and came into force in 1975.
Ans. C. Both 1 and 2 it is the only global
The Ramsar Convention environmental treaty that deals
on Wetlands is the with a particular ecosystem.
intergovernmental treaty that
provides the framework for the
1 March 2023 214

With reference to the “Tea Board” in India, consider the following statements: (UPSC
2022)
1. The Tea Board is a statutory body.
2. It is a regulatory body attached to the Ministry of Agriculture and Farmers Welfare.
3. The Tea Board’s Head Office is situated in Bengaluru.
4. The Board has overseas office at Dubai and Moscow.
Which of the statements given above are correct?
A. 1 and 3 Only It is functioning as a statutory body
of the Central Government under
B. 2 and 3 Only
the Ministry of Commerce.
C. 3 and 4 Only
Tea Board of India’s Head Office is
D. 1 and 4 Only situated in Kolkata.
Ans. D. 1 and 4 Only The Tea Board of India has
overseas offices in Moscow, Dubai
The Tea Board of India is a statutory
,Hamburg, London and New
body created under the Tea Act,
York.
1953 and it was established for the
purposes of regulating the Indian tea
industry and protecting the interests
of tea producers in India.
1 March 2023 215

Which one of the following states has launched a web


portal named “CM Da Haisi”

A. Assam
B. Manipur
C. Mizoram
D. West Bengal
Ans. B. Manipur
Manipur Chief Minister N. Biren Singh recently launched a web
portal named ‘CM Da Haisi’ (Inform to CM). It seeks to receive
complaints and grievances from the general public. The
complainants can also check the status of their complaints. The
Public Grievances Redressal and Anti-Corruption Cell at CM
Secretariat will also use the portal to collaborate with the
concerned departments.
1 March 2023 216

Recently in news Mega Science Vision (MSV)-2035 is an


initiative of

A. CSIR
B. Ministry of Science and Technology
C. DRDO
D. Office of the Principal Scientific Adviser
Ans. D. Office of the Principal Scientific Adviser
The proposed Mega Science Vision Document-2035, an
effort led by the office of the Principal Scientific Adviser
(PSA) to the government of India, aims to make students
more industry-ready and employable.
1 March 2023 217

Concerning The India-Africa Summit, consider the following statements


(UPSC 2016)
1. The third such Summit was held in 2015.
2. It was actually initiated by Jawaharlal Nehru in 1951.
Which of the statements given above is/are correct?

A. 1 Only The 2nd and 3rd summits were


B. 2 Only held in 2011 and 2015
C. Both 1 and 2 respectively.
D. Neither 1 nor 2 Recently, the 2nd India-Africa
Defence Dialogue (IADD) was
Ans. A. 1 Only held on the sidelines of
It started from 2008 with New DefExpo 2022 in Gandhinagar,
Delhi as its first venue. Since Gujarat.
then, the summit has been held
every three years, alternately in
India and Africa.
1 March 2023 218

Consider the following statements regarding National Commission for


Protection of Child Rights (NCPCR)
1. It is a constitutional body
2. It works under the Ministry of Women & Child Development.
Select the incorrect statements using the codes given below
A. 1 Only commission has a chairperson and
B. 2 Only six members of which at least two
should be women.
C. Both 1 and 2
D. Neither 1 nor 2 The Commission’s Mandate is to
ensure that all Laws, Policies,
Ans. A. 1 Only Programmes, and Administrative
NCPCR is a statutory body Mechanisms are in consonance
constituted Set up in March 2007 with the Child Rights perspective as
under Section 3 of the Commission enshrined in the Constitution of
for protection of Child Rights Act, India and also the UN Convention
2005 to protect child rights and on the Rights of the Child.
other related matters in the
Country.
It works under the Ministry of
Women &Child Development. The
1 March 2023 219

Consider the following statements


1. The Regulating Act of 1773 created the double government system.
2. The Pitt’s India Act, 1784 made lord Cornwallis as the commander-in-
chief along with the Governor-General
Select the incorrect statements using the codes given below
A. 1 Only affairs of the Company while the
B. 2 Only Board of Control was responsible
for civil, military and revenue
C. Both 1 and 2
functions of the Company .
D. Neither 1 nor 2 The Act of 1786 was enacted to
Ans. C. Both 1 and 2 extend special privileges to Lord
Cornwallis. He was made both
The dual system of government the governor general and the
was introduced by the Pitt’s India commander-in-chief. Later this
Act, 1784 where the Court of power was extended to all the
Directors of the Company were future governor generals
to look after the commercial
1 March 2023 220

Consider the following statements regarding the Morley Minto Reforms


1. It introduced bicameralism in the Indian Legislative Council at the
central level.
2. There was a system of indirect elections for the elected members of
the councils
Select the incorrect statements using the codes given below
A. 1 Only The elected members were
B. 2 Only indirectly elected by the
members of the electoral
C. Both 1 and 2
college. Local bodies elected the
D. Neither 1 nor 2 members of the electoral college,
Ans. A. 1 Only who further elected the
members of the provincial
The Montagu Chelmsford legislatures who in turn elected
reforms or the Government of the members of the central
India Act, 1919 introduced legislature
bicameralism at the central level.
1 March 2023 221

Consider the following pairs


Text Author
1. Rajatarangini Kalhan
2. Visuddhimagga Ashvagosha
3. Budhacharitra Vasumitra

Select the incorrect statements using the codes

A. 1 and 2 Only Purification was written by


B. 2 and 3 Only Buddhagosha in approximately
the 5th century under Theravada
C. 1 and 3 Only school of Buddhism.
D. 3 Only
Buddhacharita (a mahakavya)
Ans. B. 2 and 3 Only was written by Ashvagosha in
Rajatarangini was written Sanskrit about the life of Gautam
by Kalhan in Sanskrit and gives an Buddha.
account of the rulers of Kashmir.
Visuddhimagga or The path of
1 March 2023 222

Which one of the following Schedules to the Constitution


of India provides for setting up of Autonomous District
Councils?

A. 4TH Scheduled
B. 5th Scheduled
C. 6th Scheduled
D. None of the above
Ans. C. 6th Scheduled
As per the Sixth Schedule, the four states viz. Assam, Meghalaya,
Tripura and Mizoram contain the Tribal Areas which are
technically different from the Scheduled Areas (5th Schedule).
1 March 2023 223

Consider the following pairs


Lakes State
1. Renuka Manipur
2. Ropar Punjab
3. Vembanad Kerala
Select the correct pairs using the codes

A. 1 and 2 Only Vembanad lake: It is the longest


lake in India as well as the largest lake
B. 2 and 3 Only
in Kerala and is also recognized as a
C. 1 and 3 Only Ramsar site.
D. 1, 2 and 3 Renuka lake: Located in Sirmaur
Ans. B. 2 and 3 Only district of Himachal Pradesh and is
also the largest lake in Himachal
Ropar lake: It is a MAN MADE Pradesh. It has also been designated
freshwater riverine lake and also as a Ramsar site since 2005
wetland. It is located in Punjab.
1 March 2023 224

Consider the following statements regarding Wassenaar Arrangement


1. The Wassenaar Arrangement is an elite club of countries that subscribe
to arms export controls, similar to the Nuclear Suppliers Group and the
Missile Technology Control Regime.
2. India became a member of the Arrangement in 2017.
Select the correct statements using the codes given below
A. 1 only elite club of countries that subscribe
to arms export controls, similar to the
B. 2 only
Nuclear Suppliers Group and the
C. Both 1 and 2 Missile Technology Control Regime.
D. Neither 1 nor 2 The body came into being in 1996 to
Ans. C. Both 1 and 2 succeed the Cold War-era
Coordinating Committee for
India will assume the chairmanship of Multilateral Export Controls.
the Wassenaar Arrangement (WA), a
multilateral technology control India became a member of the
agreement, on 1 January 2023, for a Arrangement in 2017.
period of one year. ]
The Wassenaar Arrangement is an
1 March 2023 225

Consider the following statements regarding to the Hilton Young


Commission
1. The Royal Commission on Indian Currency and Finance was
known as the Hilton Young Commission.
2. It recommended the establishment of the Reserve Bank of India
Select the correct statements using the codes given below
A. 1 Only issues of Indian finance and
B. 2 Only currency.
C. Both 1 and 2 The Reserve Bank of India was set
up on the basis of the
D. Neither 1 nor 2 recommendations of the Royal
Ans. C. Both 1 and 2 Commission on Indian Currency
and Finance.
The Hilton Young Commission was
called the Royal Commission on
Indian Currency and Finance.
It was constituted to look into
1 March 2023 226

Consider the following statements regarding Shrinkflation


1. It stealthily boosts profit margins when companies face rising input costs.
2. It is the practice of reducing the size of a product while maintaining its
sticker price.
Select the incorrect statements using the codes given below
A. 1 Only
B. 2 Only
C. Both 1 and 2
D. Neither 1 nor 2
Ans. D. Neither 1 nor 2
Shrinkflation is the practice of reducing the size of a product while
maintaining its sticker price. It is a form of hidden inflation.
Raising the price per given amount is a strategy employed by companies,
mainly in the food and beverage industries, to stealthily boost profit
margins or maintain them in the face of rising input costs.
1 March 2023 227

Consider the following statements


1. India lies entirely in the Northern hemisphere.
2. The Tropic of Capricorn divides the country into almost two equal parts.
3. The southernmost point of the Indian Union– ‘Indira Point’ got
submerged under the sea water in 2004 during the Tsunami.
Select the correct statements using the code given below

A. 1 and 2 Only
B. 2 and 3 Only
C. 1 and 3 Only
D. 1, 2 and 3
Ans. C. 1 and 3 Only
India lies entirely in the Northern hemisphere. The Tropic of Cancer
(23° 30'N) divides the country into almost two equal parts. The
southernmost point of the Indian Union– ‘Indira Point’ got
submerged under the sea water in 2004 during the Tsunami.
1 March 2023 228

Consider the following statements


1. Viceroy Irwin accepted the demands put forward in the Delhi
Manifesto.
2. Lord Reading was the viceroy of India when Gandhi launched the civil
disobedience movement.
Select the correct statements using the codes given below

A. 1 Only of Dominion Status to India would be


discussed. The Viceroy Irwin rejected
B. 2 Only
the conditions mentioned in the Delhi
C. Both 1 and 2 Manifesto.
D. Neither 1 nor 2 Lord Irwin was the viceroy of India
Ans. D. Neither 1 nor 2 when the Civil Disobedience
Movement was started.
Delhi Manifesto of 1929 was some
conditions put forward by the Lord Reading is related to the Chauri
prominent leaders to attend the Chaura Incident and the
Round Table Conference. It was subsequent withdrawal of the Non
suggested that in the coming cooperation Movement.
Round Table Conference, the features
1 March 2023 229

Consider the following statements


1. President is the head of the state and is the highest formal authority in
the country
2. Parliament consists of the President and two Houses, Lok Sabha and
Rajya Sabha
Select the correct statements using the codes given below

A. 1 Only
B. 2 Only
C. Both 1 and 2
D. Neither 1 nor 2
Ans. C. Both 1 and 2
President is the head of the state and is the highest formal authority in the
country. Prime Minister is the head of the government and actually
exercises all governmental powers. He takes most of the decisions in the
Cabinet meetings. Parliament consists of the President and two Houses, Lok
Sabha and Rajya Sabha.
1 March 2023 230

Consider the following statements


1. William Wedderburn was the first president of Congress o foreign
origin.
2. Badruddin Tyabji was the first Muslim president of Congress.
Select the correct statements using the codes given below
A. 1 Only after George Yule at the Bombay
B. 2 Only session.
C. Both 1 and 2
Badruddin Tyabji was first Muslim
D. Neither 1 nor 2 president of the Congress and also
Ans. B. 2 Only the third president after W. C.
George Yule was the first president Bannerjee and Dadabhai Naoroji.
of Congress of foreign origin. He He presided over the Madras
was the president of the Congress session in 1889
in 1888 at the Allahabad Session.
William Wedderburn was the
president in 1889 immediately
1 March 2023 231

Consider the following statements regarding Paleolithic Age


1. Domestication of animals was started during this period.
2. Some of the famous sites of Paleolithic age are Soan valley and Potwar
Plateau on the south west of India.
Select the correct statements using the codes given below
A. 1 Only Some of the famous sites of Old
B. 2 Only Stone Age in India-
C. Both 1 and 2 1. The Soan valley and Potwar
Plateau on the northwest India.
D. Neither 1 nor 2 2. The Siwalik hills on the north
Ans. D. Neither 1 nor 2 India.
3. Bhimpetka in Madhya Pradesh.
Paleolithic or Old Stone Age sites
are generally located near water 4. Adamgarh hill in Narmada valley
sources. Domestication of animals, 5. Kurnool in Andhra Pradesh
horticulture and primitive 6. Attirampakkam near Chennai
cultivation started during
Mesolithic or Middle Stone Age.
1 March 2023 232

Consider the following statements (UPSC 2014)


1. Animal Welfare Board of India is established under the Environment
(Protection) Act, 1986.
2. National Tiger Conservation Authority is a statutory body.
3. National Ganga River Basin Authority is chaired by the Prime Minister.
Which of the statements given is/are correct?
A. 1 Only National Tiger Conservation Authority
(NTCA) is a statutory body under the
B. 2 and 3 Only
ministry of Environment, Forests and
C. 2 Only Climate Change constituted under
enabling provisions of the Wildlife
D. 1, 2 and 3
(Protection) Act, 1972, as amended in
Ans. B. 2 and 3 Only 2006, for strengthening tiger
conservation, as per powers and
Animal Welfare Board of India is a
functions assigned to it under the act.
statutory advisory body on Animal
Welfare Laws and promotes animal National Ganga River Basin Authority
welfare in the country. Established in was established in 2009 under the
1962 under Section 4 of the Prevention Environment Protection Act, 1986. It is
of Cruelty to Animals Act, 1960. Chaired by the Prime Minister of India.
1 March 2023 233

Consider the following statements regarding Preamble


1. The Supreme Court in the Minerva Mills case has held that the Preamble
is part of the Constitution.
2. The Preamble has been amended only once so far under the P.M of Mrs.
Indira Gandhi.
Select the correct statements using the codes given below
A. 1 Only
B. 2 Only
C. Both 1 and 2
D. Neither 1 nor 2
Ans. B. 2 Only
The Preamble has been amended only once so far by 42nd
Constitutional Amendment Act, 1976. At that time the Prime
Minister of India was Mrs. Indira Gandhi.
The Supreme Court in the Kesavananda Bharati case held that the
Preamble is part of the Constitution
1 March 2023 234

Rule of Law Index is released by which of the following?

A. International Court of Jurists


B. World Justice Report
C. World Economic Forum
D. None of the above
Ans. B. World Justice Report
The Rule of Law Index (ROLI) is released by the World Justice
Project (WJP), a US-based civil society group. It rates countries
based on accountability of executives and respect for fundamental
rights of people.
India has been ranked 77th out of 140 countries in 2022 by the
World Justice Project (WJP)
1 March 2023 235

Consider the following statements regarding Zonal Councils


1. The Zonal Councils are constitutional bodies established under Article
263.
2. They are only deliberative and advisory bodies.
Select the correct statements using the codes given below

A. 1 Only Reorganization Act of 1956 while the


North Eastern Council is established
B. 2 Only
by North-Eastern Council Act of 1971.
C. Both 1 and 2
Their decisions do not have a binding
D. Neither 1 nor 2 force and they are only deliberative
Ans. B. 2 Only and advisory bodies.
All the Zonal Councils are statutory
bodies established by an Act of
Parliament but not from a single Act,
the Northern, Central, Eastern,
Western and Southern Zonal Councils
are established by the State
1 March 2023 236

Consider the following statements regarding Inter-State Council


1. It is established under Article 263 as a recommendatory body.
2. In the absence of the Prime Minister, the council is presided by the
Home Minister
Select the correct statements using the codes given below
A. 1 Only establishes it as a recommendatory
body.
B. 2 Only
C. Both 1 and 2 When the prime Minister is unable to
preside over any meeting, he may
D. Neither 1 nor 2 Nominate any Union Minister of
Ans. A. 1 Only Cabinet rank to preside over the
meeting.

Article 263 of the Constitution


provides for establishment of an
Inter-State Council by the President.
The Presidential Order of 1990
1 March 2023 237

The Asurar Ali can be recognised with


which modern day road

A. GS Road
B. Assam trunk road
C. Rajgarh Road
D. Chandmari-Noonmati Road
Correct Option: C. Rajgarh Road
1 March 2023 238

Which of the following are not the federal features of the Indian
Constitution?
1.Supremacy of the Constitution
2.All India Services
3.Single Citizenship
4.Independent Judiciary
5.Bicameral Legislature
6.Integrated Judiciary

A. I, IV and V
B. I, V and VI
C. II, III and VI
D. II, III and IV
Correct Answer: C
1 March 2023 239

Which of the following Directive Principles is /are Socialistic Principle


1. To promote equal justice and to provide free legal aid to the poor
2. To promote cottage industries on co-operative basis in rural areas
3. To make provision for just and humane conditions of work
Select the correct answer from the code given below
A. 1 and 2 Only
B. 2 and 3 Only To make provision for just and
C. 1 and 3 Only humane conditions of work is
D. 1, 2 and 3 socialist principle in Article 42
Ans. C. 1 and 3 Only To promote cottage industries on
a co-operative basis in rural areas
To promote equal justice and to is a Gandhian Principle in Article
provide free legal aid to the poor. 43.
It is one of the socialist principles
provided in Article 39A of the
Constitution.
1 March 2023 240

Consider the following statements


1. BurhiDihing is a south bank tributary of Brahmaputra.
2. It flows through Dibrugarh and Sivasagar districts of Assam.
Select the correct statements using the codes given below
A. 1 Only
B. 2 Only
C. Both 1 and 2
D. Neither 1 nor 2
Ans. A. 1 Only
Dihing or Burhi Dihing is a large tributary of the Brahmaputra
River in Upper Assam. It originates in the Eastern Himalayas (the
Patkai Hills) in Arunachal Pradesh and flows through Tinsukia and
Dibrugarh Districts in Assam to its confluence with the
Brahmaputra at Dihingmukh .
1 March 2023 241

Consider the following statements about the Great Green Wall Programme
1. It was launched by the World Wide Fund for Nature.
2. It aims to promote sustainable development and climate change
mitigation.
Select the correct statements using the codes given below
A. 1 Only
B. 2 Only
C. Both 1 and 2
D. Neither 1 nor 2
Ans. B. 2 Only
It was launched in 2007 by the African Union to restore Africa’s
degraded landscapes and transform millions of lives in one of the
world’s poorest regions( the Sahel).
The project aims to restore 100 million hectares of degraded land
by 2030 .
The Great Green Wall is a symbol of hope in the face of one of the
biggest challenges of our time which is called desertification.
1 March 2023 242

Which of the following Veto Power is exercised by the President of India?


1. Absolute Veto
2. Qualified Veto
3. Suspensive Veto
4. Pocket Veto
Select the correct answer using the codes given below
A. 1, 2 and 3 Only 2. Qualified veto, which can be
overridden by the legislature with a
B. 2, 3 and 4 Only
higher majority.
C. 1, 3 and 4 Only 3. Suspensive veto, which can be
D. 1, 2, 3 and 4 overridden by the legislature with
an ordinary majority.
Ans. C. 1, 3 and 4 Only 4. Pocket veto, that is, taking no
The veto power enjoyed by the action on the bill passed by the
executive in modern states can be legislature
classified into the following four Of the above four, the President of
types: India is vested with three–
1. Absolute veto, that is, withholding absolute veto, suspensive veto and
of assent to the bill passed by pocket veto. There is no qualified
the legislature. veto in the case of Indian President
1 March 2023 243

Which of the following has launched World Report 2023?


A. World Bank
B. Human Rights Watch
C. World Economic Forum
D. International Labour Organization
Ans. B. Human Rights Watch
Recently, the 33rd edition of Human Rights Watch’s World
Report was released. Human Rights Watch (HRW) is an
international non-governmental organization
headquartered in New York City. It conducts research and
advocacy on human rights. In 1997, Human Rights Watch
shared the Nobel Peace Prize as a founding member of the
International Campaign to Ban Landmines.
1 March 2023 244

Andharubali, a place of historic importance during Ahom


Mughal war, is known for
1. Trijunction between Itakhuli, Kamakhya and Ashwaklanta
2. It was the place where battle of saraighat took place
3. History records that Haradutta Choudhury, who led the
Dundiya rebellion against the Ahom Kingdom for the
latter'soppressive occupation of Kamrup was also captured and
killedon this same sandbank in 1800
A. Only One statement is true
B. Only Two statements are true
C. All above the statements are true
D. None of the above statements are true
Ans. C. All above the statements are true
1 March 2023 245

Regarding Lachin Barphukan, select the correct statement/s


1. He Belonged to Lukhurakhun clan.
2. His first appointment was Hosotidhora Tamuli
3. His father Momai Tamuli was the first Barbarua of Ahom
administration, the post which was created by Pratap Singha in the
line of Mughal Mansabdari system
4. The first military appointment of Lachit Barphukan was Ghora
Barua.
5. At the time of appointment as Barphukan, Lachit was holding
the posting of Dolakasharia Barua to the king Chandrakanta Singha

A. 2 and 5 Only
B. 1, 3 and 5 Only
C. 2 and 4 Only
D. All of the above
Ans. D. All of the above
1 March 2023 246

Consider the following statements regarding Hema Saraswati


1. He was court poet under the King Naranarayan.
2. He was the author of Prahlada Charita.
Select the incorrect statements using the codes given below
A. 1 Only
B. 2 Only He wrote Prahlada Charita based
C. Both 1 and 2 on a story found in Vayu Purana
and Har-Gauri-Sambaad, a
D. Neither 1 nor 2 version of Kumarasambhava by
Ans. A. 1 Only Kalidasa.
He was court poet under the
patronage of Kamtapur's King
Durlabh Narayan of Kamata
Kingdom. He was the author of
Prahlada Charita.
1 March 2023 247

Consider the following statements


1. The history and philology of the Assamese language was
scientifically studied for the first time by Dr. Banikanta Kakati in
1935.
2. The first Assamese grammar, A Grammar of the Assamese
Language by William Robinson
Select the incorrect statements using the codes given
below
A. 1 Only
B. 2 Only
C. Both 1 and 2
D. Neither 1 nor 2
Ans. D. Neither 1 nor 2
Both statements are correct.
1 March 2023 248

Paskso- Monam dance is associated with which of the


following tribes of people
A. Deori
B. Khamti
C. Mising
D. None of the above
Ans. C. Mising
The Amrok is the festival observed when food from the
harvested crop is offered. A large quantity of pork meat
and chicken is also offered along with apong, and the
Paskso-Monam dances are performed by the youth with
the singing of Oi Nitom.
1 March 2023 249

Consider the following statements


1. MOITRI Scheme is associated with Police Stations renovation.
2. MOITRI stands for Mission for Overall Improvement of Thana for
Responsive Image
Select the correct statements using the codes given below
A. 1 Only Under the MOITRI Scheme, the
condition of the police stations will be
B. 2 Only
improved through the help of
C. Both 1 and 2 digitalization and thus the people of
D. Neither 1 nor 2 the state will be able to get any
information or make any complaint
Ans. C. Both 1 and 2 using the digital facilities.
The state government of Assam has The State Government of Assam will
launched a new scheme to remodel be completing the scheme in phases
the police stations of the state and to and the government will refurbish 346
ensure the police stations of the state police stations within 2023 under the
are people-friendly. The scheme has scheme. Assam government will
been named MOITRI . MOITRI stands provide a fixed sum of 2 crore INR to
for Mission of Overall Improvement of redevelop each police stations.
Thana for Responsive Image.
1 March 2023 250

Consider the following statements regarding SVAMITVA scheme


1. SVAMITVA stands for Survey of Villages and Mapping with Improvised
Technology in Village Areas.
2. It is a scheme for demarcation of rural inhabited areas using drone
technology and Continuously Operating Reference Station (CORS).
3. The scheme is piloted by the Ministry of Rural Development.
Select the correct statements using the codes given below
A. 1 and 2 Only Raj Day on 24th April 2021.
The demarcation of rural inhabited
B. 2 and 3 Only
areas would be done using Drone
C. 1 and 3 Only survey and CORS (Continuously
D. 1, 2 and 3 Operating Reference Stations)
Networks which provides mapping
Ans. A. 1 and 2 Only accuracy of 5 cms.
SVAMITVA stands for Survey of The scheme is piloted by the Ministry
Villages and Mapping with of Panchayati Raj. Survey of India is a
Improvised Technology in Village Technology Implementation Agency
Areas. It is a center sector scheme
which was launched nationally on
the occasion of National Panchayati
1 March 2023 251

Which of the following is set to launch the Jupiter ICy Moons


Explorer (Juice)?
A. NASA Venus before arriving in the Jupiter
B. ISRO system in 2031.
C. CNSA It will spend at least three years
D. ESA making detailed observations of the
giant gaseous planet Jupiter and
Ans. D. ESA three of its largest
The JUICE mission will be the first moons, Ganymede, Callisto and
time that the European Space Europa.
Agency (ESA) has sent a spacecraft It will understand if the oceans of
beyond the asteroid belt. these icy moons could have ever
JUICE is the first large-class been suitable habitats for life.
mission in the ESA Cosmic Vision
2015-2025 programme.
The spacecraft will launch on
an Ariane 5 in April 2023 on
an eight-year journey including
gravity assist flybys of Earth and
1 March 2023 252

HARIT Aaykar is an initiative of

A. National Highways Authority of India


B. Income Tax Department
C. Reserve bank of India
D. SEBI
Ans. B. Income Tax Department
The HARIT Aaykar (Hariyali Achievement Resolution by Income
Tax) initiative is launched by the Income Tax Department.
Under this initiative, the Department resolves to increase the
green cover by planting trees and creating micro-forests in and
around Income Tax Department’s buildings and other public
areas.
It was launched on the occasion of National Unity Day, 31st
October.
1 March 2023 253

Recently, PM of India has launched Mission School of Excellence in Which of


the following state?
A. Madhya Pradesh
B. Gujarat
C. Uttar Pradesh
D. None of the above
Ans. B. Gujarat
Recently, the Prime Minister of India launched Mission Schools of
Excellence at Trimandir, Adalaj, Gujarat.
Mission Schools of Excellence is a partially funded World Bank project
aimed at strengthening the education infrastructure in the State of Gujarat.
Mission Schools of Excellence is aimed at strengthening the education
infrastructure in the State through setting up of new classrooms, smart
classrooms, computer labs and overall upgradation of infrastructure in
schools.
1 March 2023 254

Samantha Cristoforetti, recently seen in news, is the European


women to command International Space station from which
country

A. Spain
B. America
C. Italy
D. None of the above
Ans. C. Italy

Italian astronaut Samantha Cristoforetti has officially become the


first European woman to command the International Space Station
(ISS). She was selected by the European Space Agency in 2009 to
become Italy’s first female astronaut.
1 March 2023 255

Which has become the country’s first completely digital


banking state recently?

A. Assam
B. Gujarat
C. Kerala
D. Tamil Nadu
Ans. C. Kerala
Kerala has become the first state in India to enable digital
banking in all eligible accounts. According to the State-
Level Bankers’ Committee (SLBC), the state now has at
least one product enabled digitally in 3.76 crore accounts.
1 March 2023 256

Recently Assam Government decided to add Bhaskarabda to official calender.


With respect to Bhaskarabda, select the correct statements
1. In addition to Vikram and Gregorian, Bhaskarabda will be used in the official
calendar by the Assam government
2. Unlike the Gregorian, where a day starts at midnight, the Assamese calendar
begins and ends at sunrise over 24 hours
3. While the Gregorian goes by the solar cycle, the Saka and Bhaskarabda use a
lunisolar system based on both the phases of the moon and the solar year.
4. The gap between Bhaskarabda and Gregorian is 539 years.
5. The gregorian year of 2023 is 1439 in Bhaskarabda
6. In the lunisolar calendar, months are lunar but years are solar.
A. Only 2, 3 and 4
B. Only 2, 3 and 6
C. 1, 2, 3, 4 and 6 Only
D. All of the above
Ans. B. Only 2, 3 and 6
1 March 2023 257

Consider the following regarding tomato flu


1. The flu affects children below five years of age.
2. This flu is noncontagious in nature.
3. The ‘tomato flu’ is caused by Coxsackie virus A 16.
Select the correct statements using the codes given below
A. 1 and 2 Only
B. 2 and 3 Only
C. 1 and 3 Only
D. All of the above
Ans. C. 1 and 3 Only
▪ Tomato flue/fever is a rare contagious disease of viral nature in humans
whose origins are not yet known.
▪ The ‘tomato flu’ is caused by Coxsackie virus A 16. It belongs to Enterovirus
family.
▪ It gets its name from the tomato-shaped red rashes that it causes on the
body of infected individuals.
▪ The flu is said to affect children below five years of age.
1 March 2023 258

Who among the following is responsible for the conduct of


elections in the Parliamentary or assembly constituency?

A. Electoral Registration Officer


B. Returning Officer
C. District Election Officer
D. None of the above
Ans. B. Returning Officer
The Returning Officer of a Parliamentary or assembly constituency
is responsible for the conduct of elections in the Parliamentary or
assembly constituency concerned. The Election Commission of
India nominates or designates an officer of the Government or a
local authority as the Returning Officer for each of the assembly
and parliamentary constituencies in consultation with the State
Government / Union Territory Administration.
1 March 2023 259

Basic Exchange and Cooperation Agreement (BECA) is an


agreement between

A. India and China


B. India and USA
C. India and Russia
D. None of the above
Ans. B. India and USA
The Basic Exchange and Cooperation Agreement mainly refer
to an agreement on the sharing of geospatial intelligence,
information on maps and satellite images for defence.
1 March 2023 260

Consider the following statements regarding Central Vigilance Commission


1. It is a constitutional body
2. Its Chairman and members are appointed by the Vice President
Select the incorrect statements using the codes given below
A. 1 Only status on the CVC.
B. 2 Only The CVC is a multi-member body
C. Both 1 and 2 consisting of a Central Vigilance
Commissioner (chairperson) and not
D. Neither 1 nor 2 more than two vigilance
Ans. C. Both 1 and 2 commissioners. They are appointed
by the President by warrant under his
It was established in 1964 by an hand and seal on the recommendation
executive resolution of the Central of a three-member committee
government. Its establishment was consisting of the prime minister as its
recommended by the Santhanam head, the Union minister of home
Committee on Prevention of affairs and the Leader of the
Corruption (1962–64). Thus, Opposition in the Lok Sabha.
originally the CVC was neither a
constitutional body nor a statutory
body. Later, in 2003, the Parliament
enacted a law conferring statutory
1 March 2023 261

World Employment and Social Outlook report: Trends 2022 was published
by
A. ILO ILO with a total 187 member
B. WB States.

C. IMF Headquarter: Geneva in


Switzerland.
D. WTO
In 1969, ILO received the Nobel
Ans. A. ILO Peace Prize for improving
ILO fraternity and peace among
nations, pursuing decent work and
It was created in 1919, as part of justice for workers, and providing
the Treaty of Versailles that ended technical assistance to other
World War I, to reflect the belief developing nations.
that universal and lasting peace
can be accomplished only if it is
based on social justice.
It became a specialized agency of
the United Nations in 1946.
India is a founding member of the
1 March 2023 262

Consider the following statements regarding Panchayati Raj


1. The Balwant Rai Mehta Committee recommended establishment of two-
tier Panchayati raj System.
2. Rajasthan was the first state to establish Panchayati Raj.
3. Ashok Mehta Committee recommended establishment of Three-tier
Panchayati raj system.
Select the incorrect statements using the codes given below
A. 1 Only raj system.
B. 1 and 2 Only Rajasthan was the first state to
C. 1 and 3 only establish Panchayati Raj. The
scheme was inaugurated by the
D. 3 Only prime minister on October 2, 1959,
in Nagaur district
Ans. C. 1 and 3 Only
The Balwant Rai Mehta Committee Ashok Mehta Committee recommended
recommended the establishment of the that The three-tier system of
scheme of ‘democratic decentralization, panchayati raj should be replaced by
which ultimately came to be known as the two-tier system.
Panchayati Raj. It recommended the
establishment of a three-tier Panchayati
1 March 2023 263

In 1871, a newspaper was published from Majuli by one Satradhikar of Vaishnavite


Satra Institution. Which Newspaper was published and by which Satradhikar of
Assam?

A. Assam Bilasini by Duttadevo Goswami


B. Asom Batori by Pitambar Goswami
C. Natun Asomiya by Narayan Goswami
D. None of the above
Correct Answer: A
In 1871, the satradhikar of Auniati Satra in
Majuli, Duttadevo Goswami, decided to bring out a newspaper
to counter the Baptist missionaries. Duttadevo Goswami
purchased a printing machine from Calcutta and ferried it to
the satra. The second newspaper of Assam, Assam Bilasini, was
launched that very year from Majuli.
1 March 2023 264

Consider the following statements and find how many statements


are correct:
1.The people revolted at Patharughat due to imposition of tax on betel nut
2.The Deputy Commissioner J. D. Anderson was killed during the revolt
3.The only memorial in the country to be constructed by the Army in
memory of peasants or civilians is at Patharughat.
4.Doli Puran is a ballad giving a vivid description of the deaths of peasants in
the firing by the British police.
5.Dineswar Sarma collected and published songs on Patharughator Ron

A. Only 1 statement is correct


B. Only 2 statements are correct
C. Only 3 statements are correct
D. Only 4 statements are correct
Only 3 Statements are correct: 3, 4 and 5.
Correct Option: C
Check more details in the description
1 March 2023 265

Tarkash 2023 is a counter Terrorism Exercise between

A. India and Russia


B. India and USA
C. India and Bangladesh
D. India and Nepal
Ans. B. India and USA
It is a counter Terrorism Exercise between India’s National
Security Guard and US Special Operations Forces.
1 March 2023 266

For which of the following books was Jayanta Madhab Bora


honoured with the prestigious Sahitya Akademi Award
A. Maghmalar Bhraman
B. Moriahola
C. Kaliloir Dintu Amar Hobo
D. Bengsata
Ans. B. Moriahola
Sahitya Akademi Award
Book Author Year
Maghmalar Bhraman Jnan Pujari 2016
Moriahola Jayanta Madhab Bora 2017
Kaliloir Dintu Amar Hobo Sananta Tanty 2018
Bengsata Apurba Kumar Saikia 2020
1 March 2023 267

Consider the following pairs reference to the Constitution of India


Subject List
1. Agriculture State list
2. Forests Union List
3. Education Concurrent List
4. Police Union List
Select the correct pairs using the codes given below
A. 1, 2 and 3 only 2. Forests
3. Weights & Measures
B. 2, 3 and 4 only
4. Protection of Wild Animals and
C. 1 and 3 only Birds
D. 1 and 4 only 5. Administration of Justice

Ans. C. 1 and 3 Only Subject List

Through the 42nd Amendment Act of Agriculture State list


1976, five subjects were transferred Forests Concurrent List
from State to Concurrent
List. They are: Education Concurrent List
1. Education Police State list
1 March 2023 268

Consider the following statements regarding Salasthamba dynasty


1. The Bargaon Copper Plates of Ratnapala states that the dynasty
of Salastambha had 21 kings.
2. The last king of Salasthamba dynasty was Tyagasimha.
Select the incorrect statements using the codes given below
A. 1 Only
B. 2 Only
C. Both 1 and 2
D. Neither 1 nor 2
Ans. D. Neither 1 nor 2
The chief of the Mlechchhas, Salastambha (c. 655-670 A.D)
laid the foundation of the rule of a new dynasty called the
Salastambhas after his name. The Bargaon Copper Plates of
Ratnapala states that the dynasty of Salastambha had 21
kings, the last one being Tyagasimha.
1 March 2023 269

Consider the following statements regarding Ratna Ojha


1. Ojha took the initiative of carrying forward the street theatre movement in
Assam.
2. He also introduced Cartoon Theater in Assam.
3. He was also production and direction the documentary film Naam-ghar.
Select the correct statements using the codes given below
A. 1 and 2 Only on the verge of fading away from the
society at that time. He also introduced
B. 2 and 3 Only
cartoon theatre in the state, has also
C. 1 and 3 Only penned several novels and poems. He
was also involved in the production and
D. 1, 2 and 3
direction the documentary film Naam-
Ans. D. 1, 2 and 3 ghar.
He was born on December 5, 1931, in He was conferred with the prestigious
Barpeta district of Assam. Ojha is well Natyasurya Award in 2005 for his
known for introducing street plays in significant contributions in the field of
Assam. After the death of his mentor theatre.
Natyasurya Phani Sarma, Ojha took the
initiative of carrying forward the street
theatre movement from 1971 in order
to save the rich art of theatre which was
1 March 2023 270

Consider the following statements regarding Chief Minister of a state


1. A person who is not a member of the state legislature cannot be
appointed as the Chief Minister.
2. He / She should be of a minimum age of 30 years to be appointed as a
chief minister of a state.
Select the incorrect statements using the codes given below

A. 1 Only ceases to be the Chief Minister.


B. 2 Only He should be a citizen of India and
C. Both 1 and 2 a minimum age of 25 years.
According to the Constitution of
D. Neither 1 nor 2 India, the Chief Minister of a state
Ans. C. Both 1 and 2 can be a member of any of the
Houses of the State legislature. The
A person who is not a member usual practice is that Chief Minister
of the state legislature can be is selected from the Lower House
appointed as Chief Minister for but he can also be selected from the
six months, within which time, Upper House also.
he should be elected to the
state legislature, failing which he
1 March 2023 271

Consider the following statements regarding Continental Drift Theory


1. The Theory was proposed by Alfred Wegener.
2. According to Wegener, all the continents formed a single
continental mass and mega ocean surrounded the same.
Select the correct statements using the codes given below
A. 1 Only of the oceans and the continents.
B. 2 Only According to Wegener’s Continental
C. Both 1 and 2 Drift theory, all the continents were
one single continental mass (called a
D. Neither 1 nor 2 Super Continent) – Pangaea and a
Ans. C. Both 1 and 2 Mega Ocean surrounded this
supercontinent. The mega ocean is
Continental drift theory was known by the name Panthalassa.
proposed by Alfred Wegener in
1912.It was first put forward by
Abraham Ortelius in 1596 before fully
being developed by Alfred Wegener.
The theory deals with the distribution
1 March 2023 272

Consider the following earthquake waves


1. S- Waves
2. P- Waves
3. Surface Waves
Select the correct order of arrival of these waves on earth’s surface in
ascending order
A. 3–2–1 These are called secondary
B. 1 – 2 – 3 waves. An important fact about
S-waves is that they can travel
C. 2 – 1 – 3 only through solid materials. The
D. 1 – 3 – 2 surface waves are the last to
report on seismograph. These
Ans. C. 2 – 1 – 3 waves are more destructive. They
P-waves move faster and are the cause displacement of rocks, and
first to arrive at the surface. hence, the collapse of structures
These are also called ‘primary occurs.
waves’. S-waves arrive at the
surface with some time lag.
1 March 2023 273

Recently which of the following country approved use of


the world’s 1st vaccine for honeybees?

A. India
B. Japan
C. USA
D. China
Ans. C. USA
The United States Department of Agriculture has
approved the first-ever vaccine for honeybees to prevent
American foulbrood disease, a fatal bacterial disease that
can destroy honeybee colonies.
1 March 2023 274

Recently India has overtaken which of the following


country to became 3rd largest auto market globally

A. Japan
B. USA
C. Russia
D. China
Ans. Japan
India has managed to overtake Japan in terms of overall
automobile sales to become the 3rd largest auto market
globally in 2022.
1 March 2023 275

Consider the following statements regarding Y20 Summit


1. Y20 is the official youth engagement group of G20.
2. India is hosting the Y20 summit for the first time.
3. The first ever Y20 Summit is to be held in India on the sidelines of the G20
Summit.
4. It provides a platform that allows youth to express their vision and ideas on
the G20 priorities.
Select the correct statements using the codes given below
A. 1, 2 and 4 Only work; climate change and disaster risk
reduction; peacebuilding and
B. 2, 3 and 4 Only
reconciliation; and youth in democracy.
C. 1 and 2 Only
D. All of the above
It provides a platform that allows youth
Ans. D. All of the above to express their vision and ideas on the
G20 priorities.
The first-ever Y20 (Youth 20) Summit is
to be held in India on the sidelines of It will focus on global youth leadership
the G20 Summit. Youth 20 (Y20) is the and partnership.
official youth engagement group of G20.
It will focus on themes of the future of
1 March 2023 276

Consider the following statements regarding Ring of Fire


1. The Ring of Fire is the result from subduction of oceanic tectonic plates
beneath lighter continental plates.
2. It is also sometimes called the circum-Pacific belt.
3. The Ring of Fire is home to more than 450 active and dormant volcanoes
forming a semicircle or horse shoe around the rim of the Pacific Ocean.
4. Most of the active volcanoes on the Ring of Fire are found on its western
edge
Select the correct statements using the codes given below
A. 1, 2 and 3 Only The Ring of Fire is home to more than
450 active and dormant volcanoes (75%
B. 2, 3 and 4 Only of Earth’s total volcanoes) forming a
C. 1 and 4 Only semicircle or horse shoe around the rim
of the Pacific Ocean.
D. All of the above
Most of the active volcanoes on the
Ans. D. All of the above Ring of Fire are found on its western
The Ring of Fire, also referred to as edge, from Russia to New Zealand.
the Circum-Pacific Belt, is a path along The Ring of Fire is the result
the Pacific Ocean characterized by from subduction of oceanic tectonic
active volcanoes and frequent plates beneath lighter continental
earthquakes. plates.
1 March 2023 277

Consider the following statements


1. Felsic rocks are rich in Silicon, Oxygen, Aluminium, Sodium and
Potassium.
2. Mafic rocks are rich in magnesium and Silicon.
3. If the rock is highly dominated by Magnesium and Iron, it is called
Ultramafic.
Select the correct statements using the codes given below
A. 1 and 2 Only
B. 2 and 3 Only
C. 1 and 3 Only
D. All of the above
Ans. C. 1 and 3 Only
Felsic and Mafic Rocks
Felsic rocks are rich in Silicon, Oxygen, Aluminium, Sodium and Potassium.
Mafic rocks are rich in magnesium and Iron.
If the rock is highly dominated by Magnesium and Iron, it is called
Ultramafic.
1 March 2023 278

Consider the following statements


1. The earthquake events are scaled either according to the magnitude or
intensity of the shock.
2. The magnitude scale is known as the Mercalli scale.
3. The intensity scale is known as the Richter scale.
4. The range of intensity scale is from 1 – 12.
Select the incorrect statements using the codes given below
A. 1 and 2 only indicates energy released during the
quake. It is expressed in absolute
B. 2 and 3 only
numbers 0-10.
C. 1 and 4 only
The intensity scale is named
D. 2 and 4 only after Mercalli, an Italian seismologist.
The intensity scale takes into account
Ans. B. 2 and 3 Only
the visible damage caused by the event.
The earthquake events are scaled either The range of intensity scale is from 1-12.
according to the magnitude or intensity
of the shock.
The magnitude scale is known as
the Richter scale. The magnitude
1 March 2023 279

Consider the following pairs


Plates Location
1. Cocos plate Between Central America and Pacific
2. Arabian plate Between South America and Pacific
Plate
3. Fuji Plate North-east of Australia

Select the correct pairs using the codes given below


A. 1 Only 2. Nazca plate- Between South
B. 1 and 2 Only America and Pacific Plate

C. 2 and 3 Only 3. Arabian plate- Mostly the Saudi


Arabian landmass
D. 1 and 3 Only
4. Philippine plate- Between the
Ans. D. 1 and 3 Only Asiatic and Pacific plate
Important minor plates- 5. Caroline plate- Between the
1. Cocos Plate-Between Central Philippine and Indian plate(North
America and Pacific of New Guinea)
6. Fuji plate- North-east of Australia
1 March 2023 280

Consider the following statements regarding National Investigation Agency


1. It was established in 2009 by NIA act 2008.
2. It is a central agency for investigate and prosecute offense which affect the
sovereignty, security and integrity of India.
Select the incorrect statements using the codes given below
A. 1 Only NIA was amended in 2019 with the
objective of speedy investigation and
B. 2 Only
prosecution of certain offences,
C. Both 1 and 2 including those committed outside
India.
D. Neither 1 nor 2
The amendment has allowed the NIA to
Ans. D. Neither 1 nor 2
investigate, in addition, cases related to:
It was established in 2009 by NIA act
• Human trafficking
2008
• Counterfeit currency or banknotes
It is a central agency for investigate and
prosecute offense which affect the • Manufacture or sale of prohibited
sovereignty, security and integrity of arms
India.
• Cyber-terrorism
It acts as the Central Counter- Terrorism
• Offences under the Explosive
Law Enforcement Agency without
Substances Act, 1908.
special permission from the states.
1 March 2023 281

The Stiglitz Commission established by the President of the United Nations


General Assembly was in the international news. The commission was
supposed to deal with: (UPSC2010)
A. The challenges posed by the impending global climate change and
prepare a road map
B. The workings of the global financial systems and to explore ways and
means to secure a more sustainable global order
C. Global terrorism and prepare a global action plan for the mitigation of
terrorism
D. Expansion of the United Nations Security Council in the present global
scenario
Ans. B
The Commission of Experts on Reforms of the International Monetary and
Financial System, called the Stiglitz Commission, was convened by the
President of the United Nations General Assembly. It was chaired by Joseph
Stiglitz.
The Commission was tasked to review the workings of the global financial
system, including major bodies such as the World Bank and the IMF, and to
suggest steps to be taken by member States to secure a more sustainable
global economic order.
1 March 2023 282

Consider the following statements regarding Central Vigilance Commission


1. It was established on the recommendation of Santhanam committee.
2. It is an independent body which is only responsible to the Parliament.
3. It is an investigating agency.
Select the correct statements using the codes given below
A. 1 and 2 Only Act conferring statutory status on the
CVC.
B. 2 and 3 Only
C. 1 and 3 Only It is an independent body which is
only responsible to the Parliament.
D. All of the above
It is not an investigating agency. The
Ans. A. 1 and 2 Only CVC either gets the investigation
The CVC was set up by the done through the CBI or through
Government in February, 1964 on the chief vigilance officers (CVO) in
recommendations of the Committee government offices.
on Prevention of Corruption, headed
by Shri K. Santhanam.
In 2003, the Parliament enacted CVC
1 March 2023 283

Consider the following statements regarding Bimbisara


1. He was the founder of Shisunaga dynasty.
2. He was a contemporary of Mahavira
3. He was the first Indian king who had a regular and standing army.
Select the correct statements using the codes given below

A. 1 and 2 Only
B. 2 and 3 Only
C. 1 and 3 Only
D. None of the above
Ans. B. 2 and 3 Only
Bimbisara
He was the founder of Haryanka dynasty.
He was a contemporary of Gautama Buddha and Mahivira
He was the first Indian king who had a regular and standing army.
1 March 2023 284

Which of the following temples are examples of Rock-Cut


Architecture ?
1. Shore temple at Mahabalipuram
2. Ladkhan temple of Aihole
3. Kailash Temple of Ellora
4. Rani ki Vav at Patan
Select the correct answer using the codes given below
A. 1 and 3 only
B. 1, 2 and 4 Only
C. 3 and 4 Only
D. All of the above
Ans. C. 3 and 4 Only
1 March 2023 285

Consider the following pairs


Monument Built by

1. Quwwat-ul-Islam Mosque Qutb-ud-din-Aibak

2. Gol Gumbaz of Bijapur Muhammad Adil Shah

3. Adhai Din Ka Jhonpra Allaudin Khilji

Select the correct pairs using the codes

A. 1 Only
B. 1 and 2 Only
C. 2 and 3 Only
D. 1, 2 and 3
Ans. B. 1 and 2 Only
1 March 2023 286

Consider the following pairs


News paper/Journal Associated Personality

1. Mirat ul Akbar Raja Rammohan Roy

2. The Hindustan Times KM Panikkar

3. Bahishkrit Bharat BR Ambedkar

Select the correct answer using the codes

A. 1 Only
B. 1 and 2 Only
C. 1 and 3 Only
D. All of the above
Ans. D. All of the above
1 March 2023 287

Consider the following pairs


Mountain Pass States

1. Jelep La Sikkim

2. Lipu Lekh Uttarakhand

3. Bom Di La Arunachal Pradesh

How many pairs given above are not correctly matched

A. Only one pair incorrect


B. Only two pairs incorrect
C. All three pairs incorrect
D. None of the above pairs are incorrect
Ans. D. None of the above pairs are incorrect
1 March 2023 288

Recently seen in News Operation Megh Chakra, is


associated with which of the following institution

A. Indian Air Force


B. CBI
C. Income Tax Department
D. Central Vigilance Commission
Ans. B. CBI
It is a pan-India drive against the circulation and sharing of
Child Sexual Abuse Material (CSAM) conducted by
the Central Bureau of Investigation (CBI)
1 March 2023 289

Which of the following is/are characteristic of ancient sect Ajivika


1. Belief in formless soul
2. Absolute determinism
3. Antinomian Ethics
Select the correct answer using the codes given below
A. 1 and 2 Only Antinomianism is a view that rejects
laws and legal aspects and encourages
B. 2 and 3 Only
going against social and moral norms.
C. 1 and 3 Only Ajivaks believe that there is no objective
moral law. Tamil Literature suggests that
D. 1, 2 and 3
Ajivaks practised Ahimsa and were non-
Ans. B. 2 and 3 Only meat eaters.
It was founded by Goshala Ajivaks had atheistic philosophy. They
Maskariputra. did not presume any deity as the
creator of the Universe. They believe
The sect is as seen as professing total
that the soul exists in every being. The
determinism in the transmigration of
soul has a material form that helps in
souls, or series of rebirths.The school is
focusing while meditating.
known for its Fate doctrine which states
that there is no free will and
everythingis predestined as a function
of cosmic principle.
1 March 2023 290

Which of the following is/are characteristic of Government of India Act,


1858
1. Rule of company was replaced by the Rule of crown.
2. Viceroy became the agent of the crown.
3. The act ended the doctrine of lapse.
4. The act also abolished the dual government of the Pitt’s India act.
Select the correct answer using the codes given below
A. 1, 2 and 3 Only Rule of crown.
B. 1, 3 and 4 Only Changed the designation of the
C. 2, 3 and 4 Only Governor-General of India to that of
Viceroy of India. Viceroy became the
D. 1, 2, 3 and 4 agent of the crown.
Ans. D. 1, 2, 3 and 4 The act ended the doctrine of lapse.
Government of India Act, 1858 The act also abolished the dual
Also known as the Act of the Good government of the Pitt’s India act.
Government of India.
Rule of company was replaced by the
1 March 2023 291

Consider the following statements regarding Sunyavada


1. It is the central doctrine of Hinayana sect of Buddhism
2. It propagates the theory that the existing world is just an illusion.
3. It has influenced Tibetan School of Buddhism
Select the correct statements using the codes given below
A. 1 and 2 Only nor the consciousness which
perceives it. According to philosophy
B. 2 and 3 Only
the constant change in the world is
C. 1 and 3 Only not real and so is the transmigration
D. 1, 2 and 3 and nirvana. This implies that reality
can be ascribed to something totally
Ans. B. 2 and 3 Only different from that is understood
Sunyavada is also known as Sunyavada philosophy is the core of
Madhyamaka. It is called the doctrine all the Tibetan School of Buddhism.
of voidness. It was founded by Santaraksita and his students
Nagarjuna and influenced the Haribhadra and Kamalashila played
evolution of Mahayana Buddhism. important roles in transmission of
According to Buddhism the world is in Sunyavada philosophy in Tibetan
a constant cosmic flux. Nagarjuna school of Buddhism.
propounded that flux cannot be real
1 March 2023 292

Consider the following statements regarding Union


Budget
1. It keeps the account of the finances of the
government for the fiscal year from 1st April to 31st
march.
2. It is mentioned under Article 112 of the India
constitution
Select the correct statements using the codes given
below
A. 1 Only
B. 2 Only
C. Both 1 and 2
D. Neither 1 nor 2
Ans. C. Both 1 and 2
1 March 2023 293

Consider the following statements


1. She served as the second women president of the
Indian National Congress.
2. She was also awarded the Kaiser-I-Hind.
Which of the following person is described in the above
statements

A. Annie Basant
B. Sarojini Naidu
C. Nellie Sengupta
D. None of the above
Ans. B. Sarojini Naidu
1 March 2023 294

Which of the following are examples of Capital Expenditure


1. Acquiring fixed and intangible assets
2. Upgrading an existing asset
3. Repairing an existing asset
4. Repayment of loan
Select the correct answer using the code given below

A. 1, 2, 3 and 4
B. 1, 2 and 4 Only
C. 2, 3 and 4 Only
D. 1, 3 and 4 Only
Ans. A. 1, 2, 3 and 4
1 March 2023 295

The Constitution of India does not contain any specific


procedure for the selection and appointment of which of
the following?
A. President
B. Vice President
C. Prime Minister
D. None of the above
Ans. C. Prime Minister
The Constitution does not contain any specific procedure for
the selection and appointment of the Prime Minister. Article 75
says only that the Prime Minister shall be appointed by the
president. However, this does not imply that the president is
free to appoint any one as the Prime Minister.
1 March 2023 296

Some time seen in news Ken-Betwa River Inter-linking


Project, will run through which of the following Tiger
reserve
A. Panna Tiger Reserve
B. Kanha Tiger Reserve
C. Pench Tiger Reserve
D. None of the above
Ans. A. Panna Tiger Reserve
Ken River passes through Panna tiger reserve. The Ken-
Betwa River Inter-Linking Project will submerge 55.78 sq
km of the Panna tiger reserve. It also involves diversion of
around 6,000 hectares of forests, mostly from the Panna
tiger reserve.
1 March 2023 297

Which of the following methods are used in contractionary monetary policy


1. Increasing the cash reserve ratio
2. Selling government securities
3. Increasing tax rates
4. Decreasing interest rates
Select the correct answer using the codes given below
A. 1 and 2 Only
B. 1 and 3 Only
C. 2, 3 and 4 Only
D. 1, 2 and 3 Only
Ans. A. 1 and 2 Only
Increasing the Cash reserve ratio can be used as part of contractionary
monetary policy. The central bank can sell government securities as part of
the open market operations (OMO) thereby sucking any extra liquidity out of
the economic system. Increasing tax rates is a part of fiscal policy, not
monetary policy. Decreasing interest rates would be a part of an
expansionary monetary policy, not a contractionary policy.
1 March 2023 298

Consider the following statements regarding Mughal Mansab system


1. “Zat” rank was an indicator of Mansabdar’s position in the imperial
hierarchy and his salary.
2. “Sawar” rank indicated the number of horsemen the Mansabdar was
required to maintain.
Select the correct statements using the codes given below
A. 1 only
B. 2 Only
C. Both 1 and 2
D. Neither 1 nor 2
Ans. C. Both 1 and 2
Mansabdari System was developed by Akbar had three important elements.
The Zat (rank), Sawar (The number of Horsemen) and Salary (paid in either
cash/Naqdi or Jagir/Watan). Sawar was the number of horsemen each
Mansabdar was required to maintain. For example Mansabdari of 10
represented a rank of 10 and maintenance of 10 sawar on part of the
Mansabdar.
1 March 2023 299

Consider the following statements regarding Wetlands


Conservation
1. Under the Ramsar convention, it is mandatory on the
part of the Government of India to protect and
conserve all the wetlands in the territory of India.
2. The wetlands (Conservation and Management) Rules,
2010 were framed by the government of India based
on the recommendations of Ramsar Convention.
Select the incorrect statements using the codes given
below

A. 1 Only Ans. C. Both 1 and 2


B. 2 Only
C. Both 1 and 2
D. Neither 1 nor 2
1 March 2023 300

Consider the following statements


1. Chairpersons of the National Human Rights Commission(NHRC) and
State Human Rights Commission (SHRC) are appointed by the
President of India.
2. Only the president of India can remove the Chairpersons of NHRC and
SHRC.
Select the incorrect statements using the codes given below

A. 1 Only
B. 2 Only
C. Both 1 and 2
D. Neither 1 nor 2
Ans. A. 1 Only
Chairpersons of SHRCs are appointed by the Governor of the State but can
be removed by the President only.
1 March 2023 301

Consider the following statements


1. The Rig Vedic society was patriarchal.
2. Both monogamy and polygamy were prevalent in the Rig Vedic
society
3. The practice of sati system and child marriage was widely
prevalent
Select the incorrect statements using the codes given below
A. 1 and 2 Only grahapathi. In Rig Vedic Society,
B. 2 only monogamy was generally practiced
while polygamy was prevalent
C. 3 only among the royal and noble
D. 1 and 3 Only families. In Rig Vedic society, there
was no child marriage and the
Ans. C. 3 Only practice of sati was absent.
The Rig Vedic society was
patriarchal. The basic unit of
society was family or graham. The
head of the family was known as
1 March 2023 302

Consider the following statements regarding All India State People’s Conference
1. It was conducted by princely states to contain the effect of congress.
2. It supported congress during the Quit India movement.
3. Jawaharlal Nehru became the President of the All-India States People’s
conference in 1939.
Select the correct statements using the codes given below
A. 1 Only convening of All India state's People
Conference. During the Quit India
B. 1 and 2 Only
movement, state people conferences
C. 2 and 3 Only formally joined the freedom struggle and
supported congress. In addition to the
D. 3 Only demand for responsible government in
Ans. C. 2 and 3 Only princely states, they asked the British to
quit India and States to be an integral part
The advancement of the national of India.
movement and an increase in political
awareness about democracy, a responsible INC supported the State People’s Struggle
government, and civil liberties were and urged the princes to introduce
responsible for organizing Praja Mandals ordemocratic representative government
state people conference in different and to grant fundamental civil rights.
princely states such as Mysore, Hyderabad, Jawaharlal Nehru became the President of
Baroda, and Kathiawar. This process came the All-India States People’s conference in
to a head in December 1927 with the 1939.
1 March 2023 303

Consider the following statements regarding Primary Deficit


1. It is the difference between revenue deficit and grants for creation of
capital assets.
2. It is measured to know the amount of borrowing that the government
can utilize, excluding the interest payments.
Select the correct statements using the codes given below

A. 1 Only expenditure of the government) and


the interest paid on the borrowings
B. 2 Only
of the previous year. Primary Deficit =
C. Both 1 and 2 Fiscal Deficit - Interest payments (on
D. Neither 1 nor 2 previous year borrowings) Primary
Deficit indicates the borrowing
Ans. B. 2 Only requirements of the government,
The primary deficit refers to the excluding interest. The primary deficit
deficit left after subtracting interest is measured to know the amount of
payments from fiscal deficit. It is the borrowing that the government can
difference between the current year’s utilize, excluding the interest
fiscal deficit (total income – total payments. It is also mentioned as a
percentage of GDP.
1 March 2023 304

Consider the following statements regarding Customs Duty


1. Customs duty is a type of Indirect tax levied only on goods imported
into India.
2. Both Centre and state government can impose customs duty.
Select the incorrect statements using the codes given below

A. 1 Only
B. 2 Only
C. Both 1 and 2
D. Neither 1 nor 2
Ans. C. Both 1 and 2
Customs Duty is a type of indirect tax levied on goods imported into India
as well as on goods exported from India. In India, the basic law for levy and
collection of customs duty is the Customs Act, 1962. It provides for levy and
collection of duty on imports and exports, import/export procedures,
prohibitions on importation and exportation of goods, penalties, offences,
etc. The Constitutional provisions have given to Union the right to legislate
and collect duties on imports and exports. The Central Board of Excise &
Customs (CBEC) is the apex body for customs matters.
1 March 2023 305

Sometime seen in news the terms Param Siddhi, Param Ganga, Pratyush are
related to
A. Student Satellites JNCASR, Bengaluru and IIT Kanpur
respectively.
B. Loan category under MUDRA
Scheme In 2020, PARAM Siddhi, the High-
Performance Computing-Artificial
C. Super Computers
Intelligence (HPC-AI) supercomputer,
D. Covid-19 Vaccine achieved global ranking of 62nd in Top
500 most powerful supercomputer
Ans. C. Super Computers
systems in the world.
India’s first supercomputer was PARAM
Pratyush and Mihir are the Fastest
8000.The National Supercomputing
Supercomputers in India. These are the
Mission (NSM) has deployed PARAM
2 High-Performance Computing Units
Ganga-a High-Performance
(HPC) with a combined speed of 6.8
Computational (HPC) facility at IIT
PetaFlops. Pratyush is established in
Roorkee, with a supercomputing
Indian Institute of Tropical Meteorology
capacity of 1.66 Petaflops.
(IITM), Pune. Mihir is established in the
PARAM Shivay, the first supercomputer National Centre for Medium-Range
assembled indigenously, was installed in Weather Forecast (NCMWF), Noida.
IIT (BHU), followed by PARAM Shakti,
PARAM Brahma, PARAM Yukti, PARAM
Sanganak at IIT-Kharagpur, IISER, Pune,
1 March 2023 306

Surya Kiran is military exercise between


A. India and Bhutan
B. India and Nepal
C. India and Bangladesh
D. None of the above
Ans. B. India and Nepal
It is a military exercise conducted annually between India
and Nepal.
15th edition- conducted at Pithoragarh in Uttarakhand.
16th Edition- conducted at Saljhandi, Nepal.
1 March 2023 307

Unakoti, famously known as the Angkor Wat of the


North-East is located in

A. Tripura
B. Assam
C. Nagaland
D. Arunachal Pradesh
Ans. A. Tripura
1 March 2023 308

Consider the following pairs


Organizations/Sabha Leaders
1. Bharat Mata Society Sardar Ajit Singh
2. Poona Sarbajanik Sabha M.G. Ranade
3. Deccan Sabha G. K. Gokhale

Select the correct pairs using the codes given below

A. 1 and 2 Only
B. 2 and 3 Only
C. 3 Only
D. 1, 2 and 3
Ans. 1, 2 and 3
1 March 2023 309

Consider the following pairs


Committee Associated
1. Aitchinson Committee Military Affairs
2. Lord Hunter Commission Jallianwala Bagh Reoprt
3. Whitley Commission Partition of Bengal
How many pairs given above is/are correctly matched
A. Only one pair is correct Committee Associated
B. Only two pairs are correct Aitchinson Public Services-
C. All three pairs are correct Committee Civil and Military
Lord Hunter Jallianwala Bagh
D. None of the above pairs Commission Reoprt
are correct
Whitley Labor Conditions
Ans. A. Only one pair is correct Commission
Peel Commission Military Affairs

Arundel Committee After Partition of


Bengal for Political
Reforms
1 March 2023 310

Consider the following statements regarding Rabindranath Tagore


1. He Presided over the Madras session of the INC in 1898.
2. He was the first non-European to receive the Nobel Prize.
3. In 1915 he was awarded knighthood by the British King George V.
Select the correct statements using the codes given below
A. 1 and 2 Only
B. 2 and 3 Only
C. 3 Only
D. 1 and 3 Only
Ans. C. 3 Only
The INC Madras Session of 1898 was presided over by Ananda Mohan Bose.
Tagore never presided congress session.
In 1913 he was awarded the Nobel Prize in Literature for his work on
Gitanjali. He was the first non-European to receive the Nobel Prize in
Literature.
In 1915 he was awarded knighthood by the British King George V. In 1919,
following the Jallianwalla Bagh massacre, he renounced his Knighthood.
1 March 2023 311

Consider the following pairs


Personality Known as
1. Rudra Baruah Luit Konwar
2. Kamalakanta Bhattacharya Agni Kobi
3. Ananda Chandra Barua Bonkonwar
Select the correct pairs using the codes given below
A. 1 Only
B. 1 and 2 Only
C. 2 and 3 Only
Personality Known as
D. 3 Only
1. Rudra Baruah Luit Konwar
2. Kamalakanta Agni Kobi
Ans. B. 1 and 2 Only
Bhattacharya
3. Ananda Chandra Barua Bakulbonor Kavi
4.Anandi Ram Das Bonkonwar
1 March 2023 312

Consider the following pairs


Personality Pen Name
1. Navakanta Barua Sima Dutta
2. Bhubanmohan Baruah Kanchan Baruah
3. Gunabhiram Barua Guru Dutta

Select the correct pairs using the codes given below

A. 1 only
B. 1 and 2 Only
C. 2 and 3 Only
D. 1, 2 and 3
Ans. D. 1, 2 and 3

You might also like